Резистор в цепи: Эта страница ещё не существует

Содержание

Резистор, для чего он нужен, где применяется в автомобилях

Сегодня мы поговорим про резистор, как основной элемент любой электрической цепи автомобиля. Для чего он нужен, какие бывают резисторы, принципы их работы, какие подходят для той или иной электрической цепи.

Эти знания могут пригодиться при ремонте автомобиля.

Три основные составляющие электрического тока

Электроэнергия достаточно плотно вошла в нашу жизнь. Используется она практически везде, и в автотранспорте в том числе.

Данный вид энергии имеет три основных составляющих – напряжение, сила тока и сопротивление.

Что касается последнего параметра, то благодаря возможности создания дополнительного сопротивления в любой точке электрической цепи можно влиять на первые два параметра.

Основным элементом для создания сопротивления является резистор. Данный элемент относится к самым востребованным, и ни одна электрическая цепь без него не обходится, и заменить его чем-либо другим не получится. А в любом автомобиле электрических цепей предостаточно.

Назначение

Основное назначение резистора – создание сопротивления для возможности контроля и регулировки силы тока и сопротивления. По сути, он является своеобразным фильтром, позволяющим на выходе из него получить электроэнергию с определенными параметрами.

Обеспечивает он все это за счет удержания тока, деления и уменьшения напряжения.

Основным параметром резистора является сопротивление, которое он создает в цепи, и измеряется оно в Омах.

Резисторы в электрической цепи автомобиля.

Именно благодаря своей функции этот элемент так часто используется в автомобилях. Ниже мы рассмотрим одни из основных составляющих авто, где используется резистор и какую конкретно функцию он там выполняет.

Система охлаждения

Итак, нагрузочный резистор используется в системе охлаждения автомобиля, а точнее, – в цепи питания вентилятора радиатора.

Стоит отметить, что раньше этот электрический элемент не использовался в данной цепи, и все работало очень просто – при достижении определенной температуры охлаждающей жидкости, температурный датчик замыкал контакты цепи питания вентилятора, и он включался в работу.

Использование же резистора позволило сделать работу электродвигателя вентилятора двух — и даже трехрежимной.

Процесс подачи питания на вентилятор при этом несколько изменился. В систему добавились также реле, а за включение вентилятора у современных авто уже отвечает электронный блок управления.

То есть, электронный блок анализирует температурные показатели датчика, и подает сигнал на реле.

В зависимости от температуры реле направляет электроэнергию по определенной цепи. Если температура охлаждающей жидкости превышена незначительно, но уже требуется ее снижение, и сигнал от ЭБУ поступил, реле направляет электроэнергию через нагрузочный резистор, который создает сопротивление, и вентилятор начинает вращаться с небольшой скоростью.

Если температура будет дальше повышаться и достигнет критической точки, реле перенаправит электроэнергию по другой цепи – в обход резистора, напрямую к вентилятору, что обеспечит его работу на полную мощность, с большой скоростью вращения.

Это схема двухрежимной работы вентилятора, которая обеспечивается наличием нагрузочного резистора в цепи. Причем она упрощенная, чтобы было более понятно.

В авто с трехрежимной работой вентилятора, принцип остается тот же, но у него уже используется два резистора – один отвечает за малые обороты вращения вентилятора, второй – за средние.

Третий же режим – аварийный, при котором вентилятор вращается с максимальной скоростью, обеспечивается за счет подачи питания на него напрямую.

Система зажигания

Второй элемент автомобиля, где можно встретить резистор – это свечи зажигания. Но далеко не все свечи оснащены им.

В конструкции данных элементов он начал появляться не так давно, и задача его заключается в подавлении радиопомех.

Кстати, сейчас ведется очень много споров, нужен ли он в свечах. Ведь резистор создает сопротивление, которое в конечном итоге влияет и на искру. А ведь чем сильнее последняя, тем лучше воспламеняется горючая смесь.

Но на самом деле на качестве искры наличие резистора сказывается незначительно, а вот на свечу – только положительно. Очень сильный искровой заряд приводит к разрушению электродов, а сопротивление снижает напряжение искры.

Но не в этом его главное назначение. Мощный искровой разряд создает достаточно сильные помехи в радиочастотном диапазоне, которые могут повлиять на работу аудиосистемы автомобиля, мобильного телефона и любого другого оборудования, чувствительного к помехам данного типа.

Интересно, что необязательно устанавливать на автомобиль свечи зажигания, оснащенные резисторами.

Дело в том, что во многих моделях шумоподавляющий элемент устанавливается в наконечники проводов высокого напряжения. Также некоторые виды самих проводов обладают достаточно неплохим сопротивлением, которого хватает для подавления радиопомех.

Резистор также может быть установлен и в бегунок трамблера, причем встречается он там на многих моделях. Его задача – та же, что и в свече зажигания или наконечнике.

Важно понимать, что во всех перечисленных элементах зажигания одновременно использоваться резисторы не могут.

При последовательном подключении этих элементов все сопротивление, которое они создают, суммируется.

То есть, если резистор будет установлен в бегунке трамблера, наконечнике, свече, то они будут создавать настолько сильное сопротивление, что значительно послабят искровой заряд, и он уже не сможет качественно воспламенять смесь. А это приведет к перебоям в работе двигателя, потере мощности, увеличению расхода топлива.

Поэтому принимать решение, стоит ли устанавливать на автомобиль свечи зажигания с резистором необходимо, тщательно ознакомившись с техдокументацией, идущей к авто.

Если изготовитель указывает, что необходимо использование таких свечей, то ими лучше пользоваться.

Читайте также:

Система обогрева салона

Еще один элемент в конструкции автомобиля, где используется резистор – система отопления салона, а точнее, – управление работой электродвигателя печки.

В любом автомобиле используется переменный резистор для изменения скорости работы электромотора обогревателя.

В нем при помощи вращающегося элемента обеспечивается возможность изменения значения сопротивления.

При включении электродвигателя на 1-ю скорость вращения, резистор обеспечивает максимальное сопротивление, при переключении на 2-ю – оно уменьшается, а при переходе на 3-ю скорость — практически полностью убирается.

 

Осветительные приборы

В последнее время резисторы стали использоваться вместе со светодиодными лампами. Данный вид ламп все больше начал применяться на авто.

Но далеко не все машины пока идут с завода, укомплектованные светодиодными осветительными приборами, а вот отдельно их купить и установить вместо штатных ламп накаливания тех же поворотников или стоп-сигналов вполне можно и многие так делают.

Но здесь возникает проблема, которая обязывает использовать резисторы.

Дело в том, что потребление электроэнергии этими лампами очень малое, из-за чего электронный блок расценивает работу светодиодов как неисправность штатной лампы.

Чтобы исправить ситуацию, используются резисторы, создающие нагрузку на линии проводки, запитывающей те осветительные приборы, в которых установлены светодиодные лампы.

В результате ЭБУ воспринимает сопротивление элемента, как работу лампы накаливания, поэтому кода ошибки не возникает.

Интересно, что при использовании таких обманок основное достоинство светодиодных ламп – малое потребление энергии, сводится к нулю, и у них остается только одно преимущество перед обычными лампами накаливания – длительный срок эксплуатации.

Виды резисторов, их особенности

Из описанных выше резисторов, которые используются в конструкции автомобиля, можно отметить два типа – нагрузочные, они же постоянные и переменные. В целом – это и есть два основных вида, которые имеют достаточно широкое применение в разных сферах.

Конечно, есть еще целый ряд всевозможных резисторов, которые отличаются по своим конструктивным особенностям. К примеру, терморезисторы, в которых сопротивление меняется от температуры, или фоторезисторы, меняющие свои параметры от освещенности. Но их мы пока касаться не будем, а рассмотрим лишь указанные два вида.

Постоянные резисторы называются так потому, что сопротивление, которое они создают – неизменное.

К примеру, если указано, что основной параметр данного элемента составляет 30 Ом, то сопротивление именно этого значения он обеспечивает и поменять его невозможно.

В переменных же резисторах сопротивление можно менять, притом вручную. Примером тому является уже упомянутое управление электродвигателем системы отопления.

К переменным резисторам относятся также подстроечные.

В таких резисторах тоже можно изменять параметр вручную, но регулировка его выполняется не в любой момент, как это делается в переменном, а лишь когда требуется перенастроить работу всей схемы, куда он включен, на длительный срок.

В автотранспорте подстроечные элементы не используются, хотя их часто можно встретить в бытовой технике.

Подбор резистора по сопротивлению

Большинство людей при выходе из строя какого-то электроприбора сдают его в ремонт или заменяют, хотя во многих случаях виноват именно резистор, тем более что он – один из самых распространенных элементов в любой схеме. Но находятся и такие, кто самостоятельно берется за ремонт.

И часто у любителей самостоятельного ремонта возникает вопрос, как правильно подобрать резистор для той или иной схемы.

Для этого возьмем простейшую схему, включающую источник питания и один потребитель.

Еще вначале было указано, что электроэнергия имеет три основные характеристики – напряжение, сила тока и сопротивление. Именно по этим параметрам и производятся все необходимые расчеты, используя для этого закон Ома.

Согласно этого закона, поскольку нам необходимо определение сопротивления, следует напряжение поделить на силу тока.

К примеру, наш источник питания обеспечивает цепь напряжением 12 В, с силой тока 0,02 А.

Чтобы определить сопротивление проводим математические расчеты – 12/0,02 и получаем сопротивление цепи 600 Ом.

Теперь непосредственно о том, как высчитать сопротивление резистора для использования в той или иной схеме. Для примера возьмем источник питания на 12 В и потребитель (лампу накаливания 3,5 В, 0,28 А).

Вначале рассчитывается сопротивление лампы – 3,5/0,28 = 12,5 Ом. Теперь узнаем, какая сила тока потечет через имеющуюся лампу – для этого берем напряжение источника питания и делим на сопротивление: 12/12,5 = 0,96 А, что в 3,5 раза превышает необходимую для работы потребителя силу тока, и если подключить потребитель, то нить лампы попросту перегорит.

Чтобы перегорания не произошло, необходимо сопротивление в цепи, равное 43,75 Ом (12,5 * 3,5). А поскольку лампа сама создает сопротивление, то в схему необходимо подключить добавочный резистор на 30 Ом. В ходе расчетов получаем – 12 В/ 42,5 Ом (сопротивление лампы и резистора) = 0,28 А.

То есть получили силу тока, необходимую для нормальной работы потребителя. В данном случае включенный в схему элемент выступил в качестве ограничителя силы тока.

Мощность рассеивания

Помимо сопротивления у резистора есть еще один немаловажный параметр – мощность рассеивания.

Любой резистор выступает своего рода ограничителем и благодаря своему сопротивлению проводит через себя только определенное напряжение и силу тока. При этом излишки, которые он не пропустил в себе не накапливает, а преобразует их в тепловую энергию и рассеивает.

Поэтому предусмотрены обозначения резисторов по мощности рассеивания.

Несоответствие данного элемента по мощности рассеивания приведет к его перегреву и разрушению. Мощность рассеивания измеряется в Ваттах.

Определить мощность рассеивания можно как по напряжению, проходящему через него, так и по силе тока.

Что касается напряжения, то формула для расчета выглядит так:

Где:

  1. Р – мощность;
  2. U – напряжение в цепи;
  3. R – сопротивление резистора.

Для расчета по силе тока формула имеет такой вид:

Где:

  1. P – мощность;
  2. I – сила тока, проходящая через резистор;
  3. R – сопротивление.

Важным условием при выборе резистора по данному параметру является то, что мощность рассеивания у него должна быть вдвое больше, чем полученная при расчетах.

К примеру, мы имеем силу тока в 0,1 А и сопротивление резистора в 100 Ом.

Исходя из формулы, получаем мощность рассеиваний в 1 Ватт (0,12 * 100 = 1), но для нормальной работы элемента выбираем резистор с мощностью рассеивания в 2 Ватт.

Отметим, что все изготавливаемые резисторы имеют строго определенное значение мощности рассеивания, что облегчает их выбор.

К тому же можно даже визуально определить, какая у резистора мощность рассеивания. Здесь все просто, чем больше по размерам элемент, тем выше значение.

Здесь мы рассмотрели резисторы – одни из самых распространенных элементов в любой электрической схеме автомобиля. Ведь они позволяют контролировать основные параметры электрической энергии благодаря воздействию всего лишь на одну из ее характеристик.

Напоследок отметим, что при расчетах необходимо следить за размерностью параметров. То есть, использовать только амперы, вольты и омы, и если указано, что сила тока составляет 20 мА, то следует перевести это значение в амперы, получив для расчетов значение в 0,02 А.

Для чего этот резистор в этой цепи?

Если переключатель разомкнут, базовое напряжение определяется прямым напряжением светодиода, например, 2 В + 0,7 В = 3,7 В. Тогда ток базы составляет (12 В — 3,7 В) / 10 кОм = 0,83 мА.

Если вы замкнете переключатель, ток через резистор 10 кОм будет разделен, чтобы проходить частично через резистор 1 кОм и частично в базу. Мы знаем, что базе требуется 3,7 В, прежде чем транзистор начнет проводить. Чтобы иметь 3,7 В, ток через 1 кОм должен быть 3,7 мА, в соответствии с законом Ома. Таким образом, если транзистор будет проводить, его базовый ток будет на 3,7 мА меньше, чем ток от источника 12 В через резистор 10 кОм.

Но мы видели, что этот ток не будет выше 0,83 мА, поэтому все будет проходить через 1 кОм, а транзистор вообще не будет работать. Поскольку он не работает, мы можем пока игнорировать его и рассчитать базовое напряжение из резисторного делителя:

ВВ= 1 к Ω1 к Ω + 10 K Ω× 12 В= 1,09 ВВВзнак равно1КΩ1КΩ+10КΩ×12Взнак равно1,09В

что на самом деле ниже требуемого 3,7 В.

Что если 1 кОм был опущен? Тогда ток заземления увеличится с 1,09 мА до 1,2 мА, и все. Эта разница в 0,1 мА не сломает банк, так что вы можете ее пропустить.

Честно говоря, я не думаю, что это хорошая трасса. Вы закрываете переключатель, чтобы выключить светодиод, а не включить, что, в общем-то, нормально, но это означает, что когда светодиод выключен, ток все равно будет течь 1,1 мА, даром. Было бы лучше разместить переключатель на стороне 10 кОм. Допустим, его функция будет инвертирована (закрытие включит светодиод), но у вас не будет тока при выключенном светодиоде. В этом случае вы все еще можете добавить резистор к земле, но его значение должно быть намного выше: 4,5 кОм потребует 0,83 мА при напряжении 3,7 В. Эти 0,83 мА были током от источника 12 В, так что это точка, в которой транзистор только начинает работать. Таким образом, значение должно быть выше, чем это. Значение 100 кОм при подключении транзистора будет составлять 37 мкА, поэтому база получит 830 мкА — 83 мкА = 750 мкА.

Если вас не волнует потеря 10%, вы можете установить резистор. Вы также можете опустить его там (не заменяя его проводом!), Тогда база будет плавать, когда переключатель разомкнут. Для биполярного транзистора это на самом деле не проблема, тем более что для его проводимости потребуется высокое 3,7 В, но для полевого МОП-транзистора этот резистор потребуется.

Обозначение мощности резистора на схеме, как её увеличить, что делать, если нет подходящего по мощности резистора

Обозначение мощности резистора на схеме, как её увеличить, что делать, если нет подходящего по мощности резистора

Резистор — пассивный элемент электрических цепей, обладающий определённым или переменным значением электрического сопротивления, предназначенный для линейного преобразования силы тока в напряжение и напряжения в силу тока, ограничения тока, поглощения электрической энергии и др. Весьма широко используемый компонент практически всех электрических и электронных устройств.

В схемах радиоэлектронной аппаратуры одним из наиболее часто встречающихся элементов является резистор, другое его название это сопротивление. У него есть целый ряд характеристик, среди которых есть мощность. В этой статье мы поговорим о резисторах, что делать, если у вас нет подходящего по мощности элемента, и почему они сгорают.

Характеристики резисторов

1. Основной параметр резистора – это номинальное сопротивление.

2. Второй параметр, по которому его выбирают – это максимальная (или предельная) рассеиваемая мощность.

3. Температурный коэффициент сопротивления – описывает, насколько изменяется сопротивление, при изменении его температуры на 1 градус Цельсия.

4. Допустимое отклонение от номинала. Обычно разброс параметров резистора от одного заявленного в пределах 5-10%, это зависит от ГОСТ или ТУ по которому он произведен, существуют и точные резисторы с отклонением до 1%, обычно стоят дороже.

5. Предельное рабочее напряжение, зависит от конструкции элемента, в бытовых электроприборах с напряжением питания 220В могут применяться практически любые резисторы.

6. Шумовые характеристики.

7. Максимальная температура окружающей среды. Это такая температура, которая может быть при достижении максимальной рассеиваемой мощности самого резистора. Об этом подробнее поговорим позже.

8. Влаго- и термоустойчивость.

Есть еще две характеристики, о которых начинающие чаще всего не знают, это:

1. Паразитная индуктивность.

2. Паразитная ёмкость.

Оба параметра зависят от типа и конструктивных особенностей резистора. Индуктивность имеет в любом проводнике, вопрос в её величины. Типовые величины паразитных индуктивностей и емкостей приводить бессмысленно. Паразитные составляющие следует учитывать при проектировании и ремонте высокочастотных приборах.

На низких частотах (например, в пределах звукового диапазона до 20 кГц), существенного влияния в работу схемы они не вносят. В высокочастотных приборах, с рабочими частотами в сотни тысяч и выше герц существенное влияние вносит даже расположение дорожек на плате и их форма.

Мощность резистора

Из курса физики многие отлично помнят формулу мощности для электричества, это: P=U*I

Отсюда следует, что она линейно зависит от тока и напряжения. Ток же через резистор зависит от его сопротивления и приложенного к нему напряжению, то есть:

I=U/R

Падение напряжения на резисторе (сколько на его выводах остаётся напряжения от приложенного к цепи, в которой он установлен), так же зависит от тока и сопротивления:

I=U/R

Теперь объясним простыми словами, что такое мощность у резистора и куда она выделяется.

У любого металла есть своё удельное сопротивление, это такая величина, которая зависит от структуры этого самого металла. Когда носители зарядов (в нашем случае электроны), под воздействием электрического тока протекают через проводник, они сталкиваются с частицами, из которого состоит металл.

В результате этих столкновений затрудняется движение тока. Если очень обобщенно сказать, то получается, так, что чем плотнее структура металла, тем сложнее протекать току (тем больше сопротивление).

На картинке пример кристаллической решетки, для наглядности.

Из-за этих столкновений выделяется тепло. Это можно представить, как если бы вы шли через толпу (большое сопротивление), где вас еще и толкают, или если бы шли по пустому коридору, где вы сильнее вспотеете?

То же самое происходит и с металлом. Мощность выделяется в виде тепла. В некоторых случаях это плохо, потому что так снижается коэффициент полезного действия прибора. В других ситуациях – это полезное свойство, например в работе ТЭНов. В лампах накаливания за счет своего сопротивления спираль раскаляется до яркого свечения.

Но как это относится к резисторам?

Дело в том, что резисторы применяют для ограничения тока при питании каких-либо устройств, или элементов цепи, или для задания режимов работы полупроводниковым приборам. Из формулы выше станет ясно, что ток снижается, за счет снижения напряжения. Лишнее напряжение можно сказать, что сгорает в виде тепла на резисторе, мощность при этом считается по той же формуле, что и общая мощность:

P=U*I

Здесь U – это количество вольт «сожженных» на резисторе, а I – это ток, который через него протекает. 2/1=144/1=144 Вт.

Всё сходится. Резистор будет выделять тепло с мощностью в 144Вт. Это условные значения, взятые в качестве примера. На практике таких резисторов вы не встретите в радиоэлектронной аппаратуре, исключением являются большие сопротивления для регулирования двигателей постоянного тока или пуска мощных синхронных машин в асинхронном режиме.

Какие бывают резисторы и как они обозначаются на схеме

Ряд мощностей резисторов стандартен: 0.05 (0.62) – 0.125 – 0.25 – 0.5 – 1 – 2 – 5

Это типовые номиналы распространенных резисторов, бывают и большие значения, или другие величины. Но этот ряд наиболее распространен. При сборке электроники используют схему электрическую принципиальную, с порядкового номера элементов. Реже указываться номинальное сопротивление, еще реже указывается номинальное сопротивление и мощность.

Чтобы быстро определить мощность резистора на схеме были введены соответствующие УГО (условные графические обозначения) по ГОСТ. Внешний вид таких обозначений и их расшифровка представлены в таблице ниже.

Вообще эти данные, а также название конкретного типа резистора указываются в перечне элементов, там же указывается и разрешенный допуск в %.

Внешне, они отличаются размером, чем мощнее элемент, тем больше его размер. Больший размер увеличивает площадь теплообмена резистора с окружающей средой. Поэтому тепло, которое выделяется при прохождении тока через сопротивление, быстрее отдаётся воздуху (если окружающая среда воздух).

Это значит, что резистор может греться с большей мощностью (выделять определенное количество тепла в единицу времени). Когда температура сопротивления достигает определенного уровня, сначала начинает выгорать внешний слой с маркировкой, дальше сгорает резистивный слой (пленка, проволока или что-то другое).

Чтобы вы оценили, как сильно может греться резистор, взгляните на нагрев спирали разобранного мощного резистора (более 5 Вт) в керамическом корпусе.

В характеристиках был такой параметр, как допустимая температура окружающей среды. Она указывается, для правильного подбора элемента. Дело в том, что раз мощность резистора ограничена способностью отдать тепло и, при этом, не перегреться, а для отдачи тепла, т.е. охлаждения элемента путем конвекции или принудительным потоком воздуха должна быть как можно большая разница температур элемента и окружающей среды.

Поэтому если вокруг элемента слишком жарко он быстрее нагреется и сгорит, даже если электрическая мощность на нем ниже максимально рассеиваемой. Нормальной температурой является 20-25 градусов Цельсия.

Что делать, если нет резистора нужной мощности?

Частой проблемой радиолюбителей является отсутствия резистора нужной мощности. Если у вас есть резисторы мощнее, чем нужно – ничего страшного в этом нет, можно ставить не задумываясь. Лишь бы он влез по размеру. Если все имеющиеся резисторы по мощности меньше, чем нужно – это уже проблема.

На самом деле решить этот вопрос достаточно просто. Вспомните законы последовательного и параллельного соединения резисторов.

1. При последовательном соединении резисторов сумма падений напряжений на всей цепочке равняется сумме падений на каждом из них. А ток, протекающий через каждый резистор равен общему току, т.е. в цепи из последовательно соединенных элементов протекает ОДИН ток, но приложенные к каждому из них напряжения РАЗНЫЕ, определяются по закону Ома для участка цепи (см. выше) Uобщ=U1+U2+U3

2. При параллельном соединении резисторов падение на всех напряжения равны, а ток, протекающий в каждой из ветвей обратно пропорционален сопротивлению ветви. Общий ток цепочки из параллельно соединенных резисторов равен сумме токов каждой из ветвей.

На этой картинке изображено всё вышесказанное, в удобной для запоминания форме.

Так, как при последовательном соединении резисторов снизится напряжение на каждом из них, а при параллельном соединении ток, то если P=U*I

Мощность, выделяемая на каждом из них, снизится соответствующим образом.

Поэтому, если у вас нет резистора 100 Ом на 1 Вт, его можно почти всегда заменить 2 резисторами на 50 Ом и 0. 5 Вт соединенными последовательно, или 2 резисторами на 200 Ом и 0.5 Вт соединенными параллельно.

Я не просто так написал «ПОЧТИ ВСЕГДА». Дело в том, что не все резисторы одинаково хорошо переносят ударные токи, в некоторых цепях, например связанные с зарядом конденсаторов большой ёмкости, в первоначальный момент времени переносят большую ударную нагрузку, которая может повредить его резистивный слой. Такие связки нужно проверять на практике или путем долгих расчетов и чтением технической документации и ТУ на резисторы, чем почти никогда и никто не занимается.

Заключение

Мощность резистора – это величина не менее важная, чем его номинальное сопротивление. Если не уделять внимания подбору сопротивлений нужно мощности, то они будут перегорать и сильно греться, что плохо в любой цепи.

При ремонте аппаратуры, особенно китайской, ни в коем случае не пытайтесь ставить резисторы меньшей мощности, лучше поставить с запасом, если есть такая возможность поместить его по габаритам на плате.

Для стабильной и надежной работы радиоэлектронного устройства нужно подбирать мощность, как минимум, с запасом в половину от предполагаемой, а лучше в 2 раза больше. Это значит, что если по расчетам на резисторе выделяется 0.9-1 Вт, то мощность резистора или их сборки должна быть не меньше, чем 1.5-2 Вт.

Ранее ЭлектроВести писали, что JinkoSolar объявила, что она установила новый рекорд эффективности для монокристаллических PERC-панелей, который составил 24,38%. Компания также разработала модуль мощностью 469,3 Вт. Кроме того, китайский производитель фотоэлектрических элементов поравнялся с фирмой Trina Solar, которая на прошлой неделе заявила о рекордном 24,58% показателе КПД монокристаллических панелей n-типа.

По материалам: electrik.info.

Переменный электрический ток. Резистор в цепи переменного тока

В идеальном колебательном контуре, то есть в контуре без активного сопротивления, возникающие электромагнитные колебания могут существовать бесконечно долго. Однако в реальных контурах всегда имеется нагрузка, обладающее сопротивлением. Поэтому часть энергии контура всегда превращается во внутреннюю энергию проводников. Проще говоря, реальные электромагнитные колебания в контуре являются затухающими. Для того чтобы они были незатухающими, необходимо компенсировались потери энергии при каждом полном колебании в контуре.

Давайте с вами вспомним, что для механических колебаний это достигалось путём воздействия внешней периодической силы. В результате в системе возникали вынужденные колебания. Аналогично этому вынужденные электромагнитные колебания в колебательном контуре происходят под действием внешней периодически изменяющейся ЭДС или внешнего изменяющегося напряжения. При этом напряжение в цепи и сила тока изменяются как по знаку, так и по модулю.

Ток, сила и направление которого периодически меняются, называется переменным.

В настоящее время основная часть электроэнергии в мире вырабатывается с помощью электромеханических индукционных генераторов переменного тока, создающими синусоидальное напряжение.

Индукционным генератором переменного тока называется устройство, предназначенное для преобразования механической энергии в энергию переменного тока.

Как следует из названия устройства, принцип действия такого генератора основан на явлении электромагнитной индукции. Основными частями индукционного генератора переменного тока являются:

индуктор — это постоянный магнит или электромагнит, который создаёт магнитное поле;

якорь — это обмотка, в которой индуцируется переменная ЭДС;

и колле́ктор — это контактные кольца и скользящие по ним контактные пластины (щётки). С помощью коллектора ток снимается или подводится к вращающимся частям.

Давайте рассмотрим принцип действия простейшего индукционного генератора на примере проводящей рамки с током, вращающейся в однородном магнитном поле с постоянной угловой скоростью.

Пусть в начальный момент времени угол между нормалью к плоскости рамки и линиями индукции магнитного поля равен нулю. Так как рамка вращается с постоянной угловой скоростью, то данный угол будет меняться с течением времени по линейному закону:

Тогда будет меняться и магнитный поток через поверхность, ограниченную плоскостью рамки:

Поскольку магнитный поток, пронизывающий рамку, изменяется со временем, то в ней согласно закону Фарадея индуцируется ЭДС индукции, равная первой производной магнитного потока по времени, взятой с обратным знаком:

 

Произведение величин, стоящих перед функцией синуса есть ничто иное, как амплитудное значение ЭДС индукции:

Отсюда следует, что изменение ЭДС индукции в контуре со временем происходит по закону синуса:

Это достаточно легко проверить, если подключить выводы вращающейся рамки к осциллографу. Нетрудно увидеть, что временная развёртка представляет собой синусоиду.

Если к выводам рамки подключить нагрузку с достаточно большим сопротивлением (намного большим, чем сопротивление рамки), то по ней будет проходить переменный ток.

По закону Ома для полной цепи его сила будет также изменяться по синусоидальному закону:

Анализируя последние два выражения, мы можем сделать вывод, что в цепи, содержащей, кроме рамки, только сопротивление, колебания напряжения и колебания силы тока совпадают по фазе, одновременно достигая максимумов и минимумов.

Однако в общем случае (например, когда в цепи присутствует конденсатор, или катушка, или то и другое одновременно) колебания силы тока в цепи и напряжения будут происходить с одинаковой частотой, но не будут совпадать по фазе:

Ещё раз обратим ваше внимание на то, что ток в цепи проходит в одном направлении в течение полуоборота рамки, а затем меняет направление на противоположное, которое также остаётся неизменным в течение следующего полуоборота.

Промежуток времени, в течение которого ЭДС совершает одно полное колебание, называется периодом переменного тока.

А число полных колебаний за одну секунду называется частотой тока.

В электрических сетях большинства стран мира (в том числе и в России) стандартная частота переменного тока равна 50 Гц. Продолжительность периода такого тока составляет всего 0,02 с. Такая частота переменного тока была выбрана с участием известного немецкого электротехника польско-русского происхождения Михаила Осиповича Доливо-Добровольского.

Однако, например, в США, Канаде и некоторых других странах по рекомендации известного сербского учёного Николы Тесла, стандартная частота переменного тока равна 60 Гц.

Мы рассмотрели на схеме принцип работы генератора переменного тока. Однако такой тип генераторов (с неподвижной магнитной системой и вращающимся якорем) используется достаточно редко. Дело в том, что при помощи подвижных контактов практически невозможно отводить от генератора ток высокого напряжения из-за сильного искрения в контактах. Поэтому почти во всех индукционных генераторах переменного тока якорь, в котором индуцируется ЭДС, устанавливают неподвижно, а вращаться заставляют индуктор.

Вращающаяся часть генератора называется ротором. Он располагается внутри неподвижной стальной станины цилиндрической формы, называемой статором. Во внутренней части статора имеются специальные пазы, в которые укладывается медный провод в виде витков. При вращении ротора в этих витках и индуцируется переменный ток.

Ротор также имеет сложную форму и представляет собой стальной сердечник с навитой на него обмоткой. По обмотке пропускается постоянный ток, который подводится через щётки и кольца от постороннего источника постоянного тока. Создаваемое этим током магнитное поле вращается вместе с ротором. При этом силовые линии поля будут пересекать проводники, вложенные в пазы статора, и индуцировать в них ЭДС.

Современные мощные генераторы вырабатывают напряжение до 15—20 кВ, а их коэффициент полезного действия может достигать 97—98 %.

Теперь давайте рассмотрим некоторые новые закономерности, которые возникают в электрической цепи при её подключении к источнику переменного тока. Итак, пусть источник создаёт переменно напряжение, изменяющееся со временем по закону синуса:

По закону Ома для участка цепи, содержащим только сопротивление, сила тока во всей цепи будет также изменяться по гармоническому закону:

Максимальные величины напряжения и силы тока называются амплитудными значениями напряжения и силы тока соответственно.

А значения напряжения и силы тока в любой момент времени называются мгновенными.

Зная их, можно рассчитать мгновенную мощность переменного тока, которая, в отличие от цепей постоянного тока, изменяется с течением времени:

Под средней за период мощностью переменного тока понимают отношение суммарной энергии, поступающей в цепь за период, к периоду.

С учётом зависимости силы тока от времени перепишем выражение для мгновенной мощности на резисторе в цепи переменного тока:

Поскольку мгновенная мощность изменяется со временем, то использовать эту величину на практике в качестве характеристики длительно протекающих процессов очень неудобно. Давайте перепишем нашу формулу для мощности немного по-другому (воспользовавшись знаниями из математики):

Как видим, в полученном уравнении первое слагаемое не зависит от времени. А второе слагаемое — это переменная составляющая, являющаяся функцией двойного угла. Её среднее значение за период (или время, кратное периоду) равно нулю, поскольку половину периода косинус принимает положительные значения, а вторую — отрицательные. Поэтому среднее значение мощности переменного тока за время, большее чем период колебаний, можно найти как половину произведения амплитудных значений силы тока и напряжения, или половину произведения квадрата амплитудного значения силы тока и сопротивления:

Таким образом, сопротивление играет двоякую роль в цепи переменного тока. Во-первых, оно ограничивает силу тока. А во-вторых, на активном сопротивлении происходит безвозвратное превращение электроэнергии в другие виды (в частности, во внутреннюю).

Выражение для средней мощности позволяет ввести действующие или эффективные значения силы тока и напряжения, которые используются в качестве основных характеристик переменного тока.

Действующее значение силы переменного тока равно силе такого постоянного тока, который, проходя в электрической цепи по активному сопротивлению, выделяет за промежуток времени, кратный периоду колебаний, такое же количество теплоты, что и данный переменный ток.

Оно численно равно квадратному корню из среднего за период значения квадрата силы переменного тока:

Аналогично можно ввести действующее значение для напряжения и ЭДС:

Амперметры и вольтметры регистрируют именно действующие значения силы тока и напряжения.

Для закрепления материала, решим с вами одну небольшую задачу. Квадратная рамка площадью 500 см2 вращается в однородном магнитном поле с индукцией 10 мТл вокруг оси, лежащей в плоскости рамки и перпендикулярной полю, совершая 25 оборотов в секунду. Определите действующее значение силы тока в рамке, если её сопротивление равно 5 Ом.

В заключение урока отметим, что закон Ома для участка цепи переменного тока, содержащего только резистор, выполняется как для амплитудных и действующих, так и для мгновенных значений напряжения и силы тока вследствие того, что их колебания совпадают по фазе.

Таким образом, резисторы оказывают сопротивление как постоянному, так и переменному току, при этом в обоих случаях в них происходит превращение электрической энергии в энергию теплового движения частиц. Вследствие этого сопротивление резисторов получило название активного или омического сопротивления.

Резисторы в цепях переменного тока

Физика > Резисторы в цепях переменного тока

 

Изучите напряжение и силу цепи переменного тока, в которую включен резистор: формула закона Ома для определения тока и напряжения цепи, схема постоянного тока.

В цепи переменного тока с резистором и источником питания используют закон Ома (V = IR).

Задача обучения

  • Используйте закон Ома для вычисления тока и напряжения.

Основные пункты

  • В переменном напряжении (V = V0sin(2πνt)) ток задается как: I =V0/R · sin(2πνt). Это выражение происходит из закона Ома: V = IR.
  • Чаще всего вместо источника постоянного тока используют переменное напряжение.
  • Рассеиваемая мощность:  P = V²0/R · sin (2πνt). Так что средняя мощность переменного тока: V²0/2R

Термин

  • Закон Ома: в электрической цепи с сопротивлениями постоянный ток выступает прямо пропорциональным напряжению.

Постоянный ток представляет собою перемещение электрического заряда в едином направлении. Это обычное состояние цепи со стабильным напряжением. Но чаще всего используют переменный ток, который периодически меняет направление. Если источник меняется с периодами (особенно синусоидально), то схему именуют цепью переменного тока. Конечно, частоты переменного тока, которые используют в домах и предприятиях, отличаются по всему миру.

(а) – Постоянные ток и напряжение остаются стабильными во времени. (b) – График напряжения и тока, основывающихся на времени, для мощности переменного тока в 60Гц. Они синусоидальные и расположены в фазе для простой схемы сопротивления. Частоты и пиковые напряжения источников сильно отличаются

Мы уже рассматривали закон Ома:

I = V/R (I – ток, V – напряжение, R – сопротивление цепи). Его можно использовать для цепей переменного и постоянного тока. Поэтому при переменном напряжении, заданном:

V = V0sin (2πνt), где V0 – пиковое напряжение, а ν – частота в герцах, ток в цепи определяется как:

I = V0/R ⋅ sin (2πνt).

В этом примере мы располагаем резистором и источником напряжения в цепи, где ток и напряжение считаются разными. Ток в резисторе перемещается назад/вперед без разности фаз, как и напряжение.

Давайте взглянем на идеальный резистор, светлеющий и тускнеющий 120 раз в секунду. Колебание светового потока говорит о колебании мощности. Так как P = IV, используем указанные выше формулы, чтобы рассмотреть зависимость мощности от времени:

P = V²0/R ⋅ sin (2πνt).

Чтобы отыскать среднюю мощность, потребляемую схемой, необходимо взять среднее время от функции:

Выходит, что:


Применение резисторов в электрических цепях: работа резистора

Резистор это один из наиболее распространенных электрических элементов, широко используемых в радиоэлектронике. Любой, кто имеет дело с электросхемами или монтажом радиодеталей на печатную плату, должен знать, для чего нужен резистор, как отличить его от других деталей (например, светодиодов), как эти компоненты ведут себя в электрических цепях.

Нелинейные резисторные изделия

Что такое резистор

Резистор что это такое? Основным свойством данного типа радиоэлементов является наличие активного сопротивления электротоку. В отличие от реактивного, оно не скапливает энергию внутри, а передает ее в окружающее пространство. Это свойство и обусловливает принцип работы резистора. В некоторых источниках и схемах слово «сопротивление» применяется в качестве наименования этой детали.

Из чего состоит резистор? Устройство этого элемента довольно простое. Основной составляющей является проволочный или пленочный компонент с большим показателем удельного сопротивления. В его роли могут выступать металлические оксиды, никелин, нихром и некоторые другие материалы.

Конструкция детали

Принцип работы

Приобретая деталь, нужно понимать, как именно работает резистор. Любой проводниковый компонент имеет определенные особенности, обусловленные его внутренним строением. Когда электроток идет по проводнику, заряженные частицы, проходя через его структуру, теряют энергетический запас, отдавая его наружу и нагревая вещество. Известно, что величина напряжения равна произведению проходящего по проводнику тока и сопротивления материала, из которого он изготовлен. Что же делает резистор? Поскольку он содержит в себе компонент с очень высокой сопротивляемостью току, при прохождении последнего на элементе понижается напряжение, и происходит выделение некоторой части мощности в виде теплоты.

Виды резисторов

При выборе подходящей детали нужно не только знать, для чего нужны в цепи резисторы, но и иметь представление о типах этих компонентов. Помимо переменных и постоянных, существуют также нелинейные приборы, чей основной параметр – сопротивление (параметр нестабилен и меняется под действием некоторого фактора внешней среды, к примеру, лучей света, температуры или напряжения).

Постоянные резисторы

Эти компоненты характеризуются неизменным значением показателя сопротивления. В отношении вариантов исполнения эти изделия бывают разными: от крупногабаритных, рассеивающих значительную мощность, до миниатюрных smd-компонентов, но все их объединяет константность сопротивления.

Изображение постоянных резисторов на схемах

Переменные резисторы

Здесь, напротив, значение сопротивления вариативно. В эту группу входят реостаты, регулирующие силу тока, и потенциометры, осуществляющие контроль напряжения. Также сюда относятся подстраивающиеся компоненты, снабженные специальными пазами. Для регуляции сопротивления в пазах надлежит проделывать манипуляции ключом, прилагающимся к прибору.

Типы переменных компонентов

Термисторы

Данные компоненты имеют в себе полупроводниковые детали и отличаются зависимостью сопротивления от окружающей температуры. Эту зависимость характеризует тепловой коэффициент, демонстрирующий, насколько меняется сопротивление элемента при перепадах температуры. У обычных термисторных изделий оно снижается при потеплении, но есть еще позисторы, чья основная характеристика при увеличении температуры также повышается.

Варисторы

Благодаря зависимости от напряжения, их широко используют для защиты сети от резких перепадов и избыточных значений упомянутого параметра. Вследствие сильного снижения сопротивления при таком инциденте ток идет через него, обходя главную цепь и обеспечивая ей изоляцию.

Важно! Из-за того, что элемент принимает на себя большую мощность, после инцидента он зачастую приходит в негодность.

Фоторезисторы

Такие компоненты меняют значение своего ключевого параметра, когда на них падает свет. Работает для этой цели, как свет солнца, так и искусственное освещение, к примеру, от фонаря.

Тензорезисторы

В них используются очень тонкие проводниковые компоненты, подвергающиеся растяжке, из-за чего их сопротивление повышается. Применяются в разного рода датчиках и электронных приборах для измерения массы.

Полупроводниковые резисторы

В таких изделиях эксплуатируются свойства тех или иных полупроводниковых материалов – менять сопротивление под действием механического давления, влажности, температуры, освещенности или иного фактора. Используемые полупроводниковые компоненты подвергаются равномерной легировке примесями. Отдельные виды последних также позволяют изготавливать разные типы изделий.

Основные характеристики

Зная, для чего в цепи нужно сопротивление, можно приступить к выбору подходящего изделия для конкретного случая. Надлежит обращать внимание на такие параметры, как номинал сопротивления и категория точности. Последняя демонстрирует процент, на который реальное сопротивление может отличаться от указанного в ту или другую сторону.

Важно! Также нужно обращать внимание на показатели выделяемой на компоненте мощности. Целесообразно приобретать изделия с мощностным запасом не менее, чем в 20%.

Где и для чего применяются

Основная область применения резисторов – контроль показателя тока. Чтобы узнать показатель ограничительного сопротивления, пользуются формулой:

R=(U2-U1)/I,

где:

  • U1 – рабочий номинал контролируемого компонента,
  • U2 – напряжение на источнике питания,
  • I – номинал тока.

Среди других областей можно отметить задание электротока транзисторам. Балластные резисторы используют для поглощения избытка напряжения.

Резистор в цепи

Детали с постоянным сопротивлениям в отечественной номенклатуре обозначаются прямоугольником, внутри которого находится определенное число черт, положение которых соответствует определенному номиналу. В зарубежных схемах их символ имеет зигзагообразную форму.

Переменные варианты отличаются направляющейся к прямоугольнику сверху линией со стрелой. Она демонстрирует опцию регуляции сопротивления. Иногда выводы элемента нумеруют цифрами.

Фоторезистор иллюстрируется прямоугольной фигурой, заключенной в круг, к которой направляется пара стрел, обозначающих световые лучи. Остальные полупроводниковые изделия символизируются зачеркнутым косой чертой прямоугольником. Буква показывает, от какого параметра зависит сопротивление (t – температура, U – напряжение и так далее).

Важно! Несколько резисторных компонентов могут быть объединены в цепь параллельно или последовательно. В первом случае будет справедливым выражение: 1/R = 1/R1+ 1/R2 + … 1/Rn. Сопротивление такой композиции будет ниже, чем у элемента с самым низким номиналом. Во втором случае итоговый показатель для системы равен сумме сопротивлений всех входящих в нее элементов.

Номиналы

Типовые значения выпускаемых в продажу резисторных элементов подчиняются некоторому ряду номиналов, в основе которого лежит положение о том, что шаг между показателями закрывает разрешенную погрешность. Например, когда номинал изделия 10 Ом, а допустимая погрешность равна 10%, у резистора, идущего в ряду последующим, будет показатель в 12 Ом. Элементы объединяют в серии, для каждой из которых существует отдельный ряд номиналов.

Маркировка

Советские изделия маркируются буквами и цифрами. При этом небольшие номиналы (до ста Ом) демонстрируются буквами R или Е, а тысячи – буквой К. Например, 250R = 250 Ом, 2К3 = 2,3 кОм = 2300 Ом, К25 = 0,25 кОм = 250 Ом. Иногда цифробуквенные коды встречаются и на импортных изделиях, например, 4W – мощность в 4 ватта, 50R – сопротивление в 50 Ом. Все-таки чаще они маркируются цветными полосами.

Цветовая маркировка

Отдельные фирмы-производители располагают разными системами значений цветовых полос. Число таковых может быть от 3 до 6. Если под рукой нет инструкции от производителя, нужно посмотреть, сколько полос имеется на корпусе элемента, и по названию фирмы найти соответствующую таблицу в сети. Первой полосой нужно считать расположенную наиболее близко к выводу.

Чтобы предохранить цепь от скачков напряжения, важно знать, что такое резистор, и уметь подбирать подходящий для конкретного случая элемент. Важно также уметь правильно рассчитать номиналы резисторов для последовательного подключения в цепь.

Видео

РЕЗИСТОР — это… Что такое РЕЗИСТОР?

  • РЕЗИСТОР — (от латинского resisto сопротивляюсь), устройство на основе проводника с нормированным постоянным (постоянный резистор) или регулируемым (переменный резистор) активным сопротивлением, используемое в электрических цепях для обеспечения требуемого… …   Современная энциклопедия

  • Резистор — (от латинского resisto сопротивляюсь), устройство на основе проводника с нормированным постоянным (постоянный резистор) или регулируемым (переменный резистор) активным сопротивлением, используемое в электрических цепях для обеспечения требуемого… …   Иллюстрированный энциклопедический словарь

  • РЕЗИСТОР — (англ. resistor от лат. resisto сопротивляюсь), радио или электротехническое изделие, основное функциональное назначение которого оказывать известное активное сопротивление электрическому току. Резистор характеризуют номинальным значением… …   Большой Энциклопедический словарь

  • резистор — резистер, варистор Словарь русских синонимов. резистор сущ., кол во синонимов: 7 • варистор (2) • …   Словарь синонимов

  • резистор — Элемент электрической цепи, предназначенный для использования его электрического сопротивления. [ГОСТ Р 52002 2003] омическое сопротивление резистор [IEV number 151 13 19] EN resistor two terminal device characterized essentially by its… …   Справочник технического переводчика

  • РЕЗИСТОР F1 — см. Рекомендуется для использования в свежем виде. Раннеспелый. Период от массовых всходов до начала технической спелости 90 100 дней. Растение низкорослое. Лист среднего размера, зеленый, слабопузырчатый, восковой налет слабый. Кочан округлый,… …   Энциклопедия семян. Овощные культуры

  • РЕЗИСТОР — радио или электротехническое изделие, обладающее только активным электрическим сопротивлением и определяющее в цепи силу тока и напряжение. Различают Р.: проволочные и непроволочные, постоянного и переменного сопротивлений класса точности от… …   Большая политехническая энциклопедия

  • Резистор — Иное название этого понятия  «Сопротивление»; см. также другие значения. Шесть резисторов разных номиналов и точности, промаркированные с помощью цветовой схемы Резистор …   Википедия

  • резистор — а; м. [англ. resistor] Радио или электротехническая деталь, создающая сопротивление в электрической цепи. * * * резистор (англ. resistor, от лат. resisto  сопротивляюсь), радио или электротехническое изделие, основное функциональное назначение… …   Энциклопедический словарь

  • Резистор — 1 . Резистор D. Widerstand E. Resistor F. Résistance По ГОСТ 19880 74* Источник: ГОСТ 21414 75: Резисторы. Термины и определения оригинал документа …   Словарь-справочник терминов нормативно-технической документации

  • Учебное пособие по физике: Параллельные схемы

    Как упоминалось в предыдущем разделе Урока 4, два или более электрических устройства в цепи могут быть соединены последовательным или параллельным соединением. Когда все устройства соединены с использованием параллельных соединений, схема называется параллельной схемой . В параллельной схеме каждое устройство помещается в свою отдельную ветвь . Наличие ответвлений означает, что существует несколько путей, по которым заряд может проходить через внешнюю цепь.Каждый заряд, проходящий через контур внешней цепи, будет проходить через единственный резистор, присутствующий в одной ветви. Прибыв в место разветвления или узел, плата выбирает , через какую ветвь пройти на обратном пути к терминалу с низким потенциалом.

    Краткое сравнение и контраст между последовательными и параллельными цепями было сделано в предыдущем разделе Урока 4. В этом разделе было подчеркнуто, что добавление большего количества резисторов в параллельную цепь приводит к довольно неожиданному результату — уменьшению общего сопротивления. .Поскольку существует несколько путей, по которым может протекать заряд, добавление еще одного резистора в отдельную ветвь обеспечивает еще один путь, по которому заряд может проходить через основную область сопротивления в цепи. Это уменьшенное сопротивление в результате увеличения количества ветвей будет иметь эффект увеличения скорости, с которой течет заряд (также известной как ток). Чтобы сделать этот довольно неожиданный результат более разумным, была введена аналогия с платными дорогами. Плата за проезд — это основное место сопротивления автомобильному потоку на платной дороге.Добавление дополнительных пунктов сбора платы за проезд в пределах их собственного отделения на платной дороге обеспечит больше путей для автомобилей, проезжающих через станцию ​​сбора платы за проезд. Эти дополнительные пункты пропуска снизят общее сопротивление потоку автомобилей и увеличат скорость их движения.

    Текущая

    Скорость, с которой заряд проходит через цепь, называется током. Заряд НЕ накапливается и не начинает накапливаться в любом заданном месте, так что ток в одном месте больше, чем в других местах.Заряд НЕ расходуется резисторами таким образом, что в одном месте ток меньше, чем в другом. В параллельной схеме заряд делит на отдельные ветви, так что в одной ветви может быть больше тока, чем в другой. Тем не менее, если брать в целом, общая сумма тока во всех ветвях при сложении равна величине тока в местах за пределами ветвей. Правило, что ток везде один и тот же , все еще работает, только с изюминкой.Сила тока вне ветвей равна сумме токов в отдельных ветвях. Это все еще та же величина тока, только разделенная на несколько путей.

    В форме уравнения этот принцип можно записать как

    I всего = I 1 + I 2 + I 3 + …

    , где I всего — это общая сумма тока вне ветвей (и в батарее), а I 1 , I 2 и I 3 представляют ток в отдельных ветвях цепи.

    В этом блоке широко использовалась аналогия между расходом заряда и расходом воды. Еще раз вернемся к аналогии, чтобы проиллюстрировать, как сумма текущих значений в ветвях равна сумме вне ветвей. Поток заряда в проводах аналогичен потоку воды в трубах. Рассмотрим приведенные ниже схемы, на которых поток воды в трубах делится на отдельные ответвления. В каждом узле (место разветвления) вода проходит двумя или более отдельными путями.Скорость, с которой вода поступает в узел (измеряется в галлонах в минуту), будет равна сумме расходов в отдельных ветвях за пределами узла. Точно так же, когда две или более ветви подаются в узел, скорость, с которой вода вытекает из узла, будет равна сумме расходов в отдельных ветвях, которые подаются в узел.

    Тот же принцип разделения потока применяется к электрическим цепям. Скорость, с которой заряд поступает в узел, равна сумме расходов в отдельных ветвях за пределами узла.Это проиллюстрировано в приведенных ниже примерах. В примерах вводится новый символ схемы — буква A, заключенная в круг. Это символ амперметра — устройства, используемого для измерения силы тока в определенной точке. Амперметр способен измерять ток, оказывая при этом незначительное сопротивление потоку заряда.

    Диаграмма A показывает два резистора, включенных параллельно с узлами в точках A и B. Заряд течет в точку A со скоростью 6 ампер и делится на два пути — один через резистор 1, а другой — через резистор 2.Ток в ветви с резистором 1 составляет 2 ампера, а ток в ветви с резистором 2 — 4 ампера. После того, как эти две ветви снова встретятся в точке B, чтобы сформировать единую линию, ток снова станет равным 6 ампер. Таким образом, мы видим, что принцип, согласно которому ток вне ветвей равен сумме тока в отдельных ветвях, справедлив.

    I итого = I 1 + I 2

    6 ампер = 2 ампера + 4 ампера

    Схема B выше может быть немного сложнее, если три резистора расположены параллельно.На схеме обозначены четыре узла, обозначенные буквами A, B, C и D. Заряд течет в точку A со скоростью 12 ампер и делится на два пути: один проходит через резистор 1, а другой направляется к точке B (и резисторам 2). и 3). 12 ампер тока делятся на 2-амперную (через резистор 1) и 10-амперную (в направлении точки B). В точке B происходит дальнейшее разделение потока на два пути — один через резистор 2, а другой через резистор 3. Ток в 10 ампер, приближающийся к точке B, делится на 6-амперный канал (через резистор 2) и 4-канальный. -амперный тракт (через резистор 3).Таким образом, видно, что значения тока в трех ветвях составляют 2 ампера, 6 ампер и 4 ампера, и что сумма значений тока в отдельных ветвях равна току вне ветвей.

    I всего = I 1 + I 2 + I 3

    12 ампер = 2 ампер + 6 ампер + 4 ампер

    Также может быть проведен анализ потока в точках C и D, и будет замечено, что сумма расходов потока в этих точках равна скорости потока, находящейся непосредственно за этими точками.

    Эквивалентное сопротивление

    Фактическая величина тока всегда изменяется обратно пропорционально величине общего сопротивления. Существует четкая взаимосвязь между сопротивлением отдельных резисторов и общим сопротивлением набора резисторов. Чтобы исследовать эту взаимосвязь, давайте начнем с простейшего случая, когда два резистора помещены в параллельные ветви, каждый из которых имеет одинаковое значение сопротивления 4 Ом.Поскольку схема предлагает два равных путей для потока заряда, только половина заряда выберет для прохождения через данную ветвь. В то время как каждая отдельная ветвь предлагает сопротивление 4 Ом любому заряду, который проходит через нее, только половина всего заряда, протекающего по цепи, будет встречать сопротивление 4 Ом этой отдельной ветви. Таким образом, что касается аккумулятора, который накачивает заряд, наличие двух параллельно подключенных резисторов 4 Ом было бы эквивалентно наличию одного резистора 2 Ом в цепи.Таким же образом, наличие двух параллельно подключенных резисторов сопротивлением 6 Ом было бы эквивалентно наличию в цепи одного резистора сопротивлением 3 Ом. А наличие двух параллельных резисторов 12 Ом было бы эквивалентно наличию в цепи одного резистора 6 Ом.

    Теперь давайте рассмотрим другой простой случай, когда параллельно соединены три резистора, каждый из которых имеет одинаковое сопротивление 6 Ом. При трех равных путях прохождения заряда через внешнюю цепь только одна треть заряда будет проходить через данную ветвь.Каждая отдельная ветвь обеспечивает сопротивление 6 Ом проходящему через нее заряду. Однако тот факт, что только одна треть заряда проходит через определенную ветвь, означает, что общее сопротивление цепи составляет 2 Ом. Что касается батареи, которая нагнетает заряд, наличие трех параллельных резисторов 6 Ом было бы эквивалентно наличию одного резистора 2 Ом в цепи. Таким же образом, наличие трех параллельно подключенных резисторов сопротивлением 9 Ом было бы эквивалентно наличию в цепи одного резистора сопротивлением 3 Ом.А наличие трех параллельных резисторов 12 Ом было бы эквивалентно наличию одного резистора 4 Ом в цепи.

    Это концепция эквивалентного сопротивления. Эквивалентное сопротивление цепи — это величина сопротивления, которая потребуется одному резистору, чтобы сравняться с общим эффектом от набора резисторов, присутствующих в цепи. Для параллельных цепей математическая формула для вычисления эквивалентного сопротивления ( R eq ) составляет

    . 1/ экв. = 1/ 1 + 1/ 2 + 1/ 3 +…

    , где R 1 , R 2 и R 3 — значения сопротивления отдельных резисторов, подключенных параллельно. Приведенные выше примеры можно рассматривать как простые случаи, в которых все пути обладают одинаковым сопротивлением отдельному заряду, который проходит через них. Приведенные выше простые случаи были выполнены без использования уравнения. Тем не менее, уравнение подходит как для простых случаев, когда резисторы ответвления имеют одинаковые значения сопротивления, так и для более сложных случаев, когда резисторы ответвления имеют разные значения сопротивления.Например, рассмотрим применение уравнения к одному простому и одному сложному случаю ниже.

    Случай 1 : три резистора 12 Ом включены параллельно

    1 / R экв = 1 / R 1 + 1 / R 2 + 1 / R 3

    1 / R экв = 1 / (12 Ом) + 1 / (12 Ом) + 1 / (12 Ом)

    Использование калькулятора …

    1 / R экв. = 0,25 Ом -1

    R экв = 1 / (0,25 Ом -1 )

    R экв = 4,0 Ом

    Случай 2 : резисторы 5,0 Ом, 7,0 Ом и 12 Ом включены параллельно

    1 / R экв = 1 / R 1 + 1 / R 2 + 1 / R 3

    1 / R экв = 1 / (5.0 Ом) + 1 / (7,0 Ом) + 1 / (12 Ом)

    Использование калькулятора …

    1 / R экв = 0,42619 Ом-1

    R экв = 1 / (0,42619 Ом -1 )

    R экв. = 2,3 Ом


    Ваша очередь попробовать

    Нужно больше практики? Используйте два параллельных резистора виджет ниже, чтобы попробовать некоторые дополнительные проблемы.Введите любые два желаемых значения сопротивления. Воспользуйтесь калькулятором, чтобы определить значения рэндов экв. . Затем нажмите кнопку Отправить , чтобы проверить свои ответы. Попробуйте столько раз, сколько хотите, с разными значениями сопротивления.

    Падения напряжения для параллельных ветвей

    В разделе «Схемы» учебного пособия «Физический класс» подчеркивалось, что любое повышение напряжения, полученное за счет заряда в батарее, теряется из-за заряда, когда он проходит через резисторы внешней цепи.Общее падение напряжения во внешней цепи равно увеличению напряжения при прохождении заряда через внутреннюю цепь. В параллельной схеме заряд не проходит через каждый резистор; скорее, он проходит через единственный резистор. Таким образом, полное падение напряжения на этом резисторе должно соответствовать напряжению батареи. Не имеет значения, проходит ли заряд через резистор 1, резистор 2 или резистор 3, падение напряжения на резисторе, которое выбирает для прохождения , должно равняться напряжению батареи.В форме уравнения этот принцип можно было бы выразить как

    В аккумулятор = В 1 = В 2 = В 3 = …

    Если три резистора размещены в параллельных ветвях и питаются от 12-вольтовой батареи, то падение напряжения на каждом из трех резисторов составляет 12 вольт. Заряд, протекающий по цепи, встретит только один из этих трех резисторов и, таким образом, столкнется с одним падением напряжения на 12 вольт.

    Диаграммы электрических потенциалов были представлены в Уроке 1 этого устройства и впоследствии использовались для иллюстрации последовательных падений напряжения, происходящих в последовательных цепях.Диаграмма электрического потенциала — это концептуальный инструмент для представления разности электрических потенциалов между несколькими точками электрической цепи. Рассмотрим приведенную ниже принципиальную схему и соответствующую диаграмму электрических потенциалов.

    Как показано на диаграмме электрических потенциалов, все позиции A, B, C, E и G имеют высокий электрический потенциал. Один заряд выбирает только один из трех возможных путей; таким образом, в позиции B один заряд будет двигаться к точкам C, E или G, а затем пройдет через резистор, находящийся в этой ветви.Заряд не теряет свой высокий потенциал до тех пор, пока он не пройдет через резистор, либо от C к D, от E к F или от G к H. После того, как он проходит через резистор, заряд возвращается почти до 0 вольт и возвращается к отрицательному. клемму аккумуляторной батареи для повышения ее напряжения. В отличие от последовательных цепей, заряд в параллельной цепи встречает единственное падение напряжения на своем пути через внешнюю цепь.

    Ток через заданную ветвь можно предсказать, используя уравнение закона Ома, падение напряжения на резисторе и сопротивление резистора.Поскольку падение напряжения на каждом резисторе одинаково, фактором, определяющим, что резистор имеет наибольший ток, является сопротивление. Резистор с наибольшим сопротивлением испытывает наименьший ток, а резистор с наименьшим сопротивлением — наибольший ток. В этом смысле можно сказать, что заряд (как и люди) выбирает путь наименьшего сопротивления. В форме уравнения это может быть указано как

    I 1 = Δ V 1 / R 1 I 2 = Δ V 2 / R 2 I 3 = Δ V 3 / R 3

    Этот принцип иллюстрируется схемой, показанной ниже.Произведение I • R одинаково для каждого резистора (и равно напряжению батареи). Тем не менее, ток у каждого резистора разный. Ток наибольший там, где сопротивление наименьшее, и ток наименьший, где сопротивление наибольшее.

    Математический анализ параллельных цепей

    Приведенные выше принципы и формулы могут использоваться для анализа параллельной цепи и определения значений тока и разности электрических потенциалов на каждом из резисторов в параллельной цепи.Их использование будет продемонстрировано математическим анализом схемы, показанной ниже. Цель состоит в том, чтобы использовать формулы для определения эквивалентного сопротивления цепи (R eq ), тока через батарею (I до ), а также падений напряжения и тока для каждого из трех резисторов.

    Анализ начинается с использования значений сопротивления отдельных резисторов для определения эквивалентного сопротивления цепи.

    1 / R экв = 1 / R 1 + 1 / R 2 + 1 / R 3 = (1/17 Ом) + (1/12 Ом) + (1/11 Ом)

    1 / R экв = 0.23306 Ом -1

    R экв = 1 / (0,23306 Ом -1 )

    R экв = 4,29 Ом

    (округлено от 4,29063 Ом)

    Теперь, когда известно эквивалентное сопротивление, ток в батарее можно определить с помощью уравнения закона Ома. При использовании уравнения закона Ома (ΔV = I • R) для определения тока в батарее важно использовать напряжение батареи для ΔV и эквивалентное сопротивление для R.Расчет показан здесь:

    I до = ΔV аккумулятор / R eq = (60 В) / (4,29063 Ом)

    I до = 14,0 А

    (округлено от 13,98396 А)

    Напряжение батареи 60 В представляет собой усиление электрического потенциала за счет заряда, проходящего через батарею. Заряд теряет такое же количество электрического потенциала при любом прохождении через внешнюю цепь.То есть падение напряжения на каждом из трех резисторов такое же, как и напряжение, полученное в батарее:

    ΔV аккумулятор = ΔV 1 = ΔV 2 = ΔV 3 = 60 В

    Осталось определить три значения — ток каждого отдельного резистора. Закон Ома снова используется для определения значений тока для каждого резистора — это просто падение напряжения на каждом резисторе (60 В), деленное на сопротивление каждого резистора (указанное в формулировке задачи).Расчеты показаны ниже.

    I 1 = ΔV 1 / R 1

    I 1 = (60 В) / (17 Ом)

    I 1 = 3,53 А

    I 2 = ΔV 2 / R 2

    I 2 = (60 В) / (12 Ом)

    I 2 = 5,00 А

    I 3 = ΔV 3 / R 3

    I 3 = (60 В) / (11 Ом)

    Я 3 = 5.45 ампер

    Для проверки точности выполненных математических расчетов целесообразно проверить, удовлетворяют ли вычисленные значения принципу, согласно которому сумма значений тока для каждого отдельного резистора равна общему току в цепи (или в батарее). . Другими словами, I to = I 1 + I 2 + I 3 ?

    Является ли I tot = I 1 + I 2 + I 3 ?

    Из 14.0 ампер = 3,53 ампер + 5,00 ампер + 5,45 ампер?

    14,0 А = 13,98 А?

    Да !!

    (Разница в 0,02 ампера — это просто результат предыдущего округления значения I до от 13,98.)

    Математический анализ этой параллельной цепи включал смесь концепций и уравнений. Как это часто бывает в физике, отделение понятий от уравнений при принятии решения физической проблемы является опасным актом.Здесь необходимо учитывать концепции, согласно которым падение напряжения на каждом из трех резисторов равно напряжению батареи, и что сумма тока в каждом резисторе равна общему току. Эти представления необходимы для завершения математического анализа. В следующей части Урока 4 будут исследованы комбинированные или составные схемы, в которых одни устройства включены параллельно, а другие — последовательно.

    Создавайте, решайте и проверяйте свои собственные проблемы с помощью виджета Equivalent Resistance ниже.Создайте себе проблему с любым количеством резисторов и любыми номиналами. Решать проблему; затем нажмите кнопку «Отправить», чтобы проверить свой ответ.

    Мы хотели бы предложить … Зачем просто читать об этом и когда можно с этим взаимодействовать? Взаимодействовать — это именно то, что вы делаете, когда используете одно из интерактивных материалов The Physics Classroom. Мы хотели бы предложить вам совместить чтение этой страницы с использованием нашего интерактивного средства построения цепей постоянного тока.Вы можете найти его в разделе Physics Interactives на нашем сайте. Построитель цепей постоянного тока предоставляет учащемуся набор для построения виртуальных цепей. Вы можете легко перетащить источники напряжения, резисторы и провода на рабочее место, а также расположить и подключить их так, как захотите. Вольтметры и амперметры позволяют измерять падение тока и напряжения. Нажатие на резистор или источник напряжения позволяет изменять сопротивление или входное напряжение. Это просто. Это весело. И это безопасно (если вы не используете его в ванне).


    Проверьте свое понимание

    1. По мере того, как в цепь добавляется все больше и больше резисторов, эквивалентное сопротивление цепи ____________ (увеличивается, уменьшается), а общий ток цепи ____________ (увеличивается, уменьшается).

    2.Три одинаковых лампочки подключены к D-ячейке, как показано ниже. P, Q, X, Y и Z обозначают местоположения вдоль цепи. Какое из следующих утверждений верно?

    а. Ток в точке Y больше, чем ток в точке Q.

    г. Ток на Y больше, чем на P.

    .

    г. Ток в точке Y больше, чем ток в точке Z.

    г. Ток в точке P больше, чем ток в точке Q.

    .

    e.Ток на Q больше, чем на P.

    ф. Сила тока одинакова во всех местах.

    3. Три одинаковые лампочки подключены к D-ячейке, как показано ниже. P, Q, X, Y и Z обозначают местоположения вдоль цепи. В каком (а) месте (ах), если таковые имеются, будет ток …

    а. … так же, как у X?

    г…. такой же, как у Q?

    г. … так же, как у Y?

    г. … меньше, чем у Q?

    e. … меньше, чем у P?

    ф. … вдвое больше, чем у Z?

    г. … в три раза больше, чем в Y?

    4. Какие изменения можно внести в схему ниже, чтобы уменьшить ток в ячейке? Перечислите все подходящие варианты.

    а. Увеличьте сопротивление лампы X.

    г. Уменьшите сопротивление лампы X.

    г. Увеличьте сопротивление лампы Z.

    г. Уменьшите сопротивление лампы Z.

    .

    e. Увеличьте напряжение ячейки (как-нибудь).

    ф. Уменьшите напряжение ячейки (как-нибудь).

    г. Снять лампу Y.

    .

    5.Аккумулятор на 12 В, резистор на 12 Ом и резистор на 4 Ом подключаются, как показано на рисунке. Ток в резисторе 12 Ом равен ____ току в резисторе 4 Ом.

    а. 1/3

    г. 1/2

    г. 2/3

    г. то же, что

    e.1,5 раза

    ф. дважды

    г. трижды

    ч. четыре раза


    6. Аккумулятор на 12 В, резистор на 12 Ом и резистор на 4 Ом подключены, как показано.Падение напряжения на резисторе 12 Ом равно ____ падению напряжения на резисторе 4 Ом.

    а. 1/3

    г. 1/2

    г. 2/3

    г. то же, что

    e. 1,5 раза

    ф.дважды

    г. трижды

    ч. четыре раза

    7. Аккумулятор на 12 В и резистор на 12 Ом подключаются, как показано на схеме. Резистор на 6 Ом добавлен к резистору на 12 Ом, чтобы создать цепь Y, как показано.Падение напряжения на резисторе 6 Ом в цепи Y равно ____ падению напряжения на резисторе в цепи X.

    а. больше, чем

    г. меньше

    г. то же, что

    8. Используйте свое понимание эквивалентного сопротивления, чтобы заполнить следующие утверждения:

    а. Два резистора сопротивлением 6 Ом, помещенные параллельно, обеспечат сопротивление, эквивалентное сопротивлению одного резистора _____ Ом.

    г. Три резистора 6 Ом, помещенные параллельно, обеспечат сопротивление, эквивалентное одному резистору _____ Ом.

    г. Три резистора сопротивлением 8 Ом, помещенные параллельно, обеспечат сопротивление, эквивалентное сопротивлению одного резистора _____ Ом.

    г. Три резистора с сопротивлением 2 Ом, 4 Ом и 6 Ом размещены параллельно. Они обеспечили бы сопротивление, эквивалентное одному резистору _____ Ом.

    e. Три резистора с сопротивлением 5 Ом, 6 Ом и 7 Ом размещены параллельно.Они обеспечили бы сопротивление, эквивалентное одному резистору _____ Ом.

    ф. Три резистора с сопротивлением 12 Ом, 6 Ом и 21 Ом размещены параллельно. Они обеспечили бы сопротивление, эквивалентное одному резистору _____ Ом.

    9. На основании ваших ответов на вышеуказанный вопрос заполните следующую формулировку:

    Общее или эквивалентное сопротивление трех параллельно включенных резисторов будет _____.

    а. больше, чем сопротивление самого большого значения R.

    г. меньше, чем сопротивление наименьшего значения R из трех.

    г. где-то между наименьшим значением R и наибольшим значением R.

    г. … ерунда! Такого обобщения сделать нельзя. Результаты меняются.

    10. Три резистора включены параллельно.При размещении в цепи с источником питания 12 В. Определите эквивалентное сопротивление, полный ток цепи, падение напряжения и ток в каждом резисторе.


    17,5 Параллельные резисторы | Электрические цепи

    17,5 Резисторы параллельные (ЭСАФК)

    Когда мы добавляем резисторы параллельно цепи:

    • Есть больше путей для прохождения тока, что обеспечивает разделение тока по разным путям .

    • Напряжение на резисторах то же . Напряжение на батарее в цепи равно напряжению на каждом из параллельных резисторов:

      \ [{V} _ {\ text {battery}} = {V} _ {1} = {V} _ {2} = {V} _ {3} \ ldots \]
    • Сопротивление току уменьшается . Общее сопротивление \ ({R} _ {P} \) дается выражением:

      \ [\ frac {1} {{R} _ {P}} = \ frac {1} {{R} _ {1}} + \ frac {1} {{R} _ {2}} + \ ldots \ ]

    При параллельном подключении резисторов начальная и конечная точки для всех резисторов одинаковы.Эти точки имеют одинаковую потенциальную энергию, поэтому разность потенциалов между ними одинакова, независимо от того, что между ними вставлено. Между двумя точками может быть один, два или несколько резисторов, разность потенциалов не изменится. Вы можете игнорировать любые компоненты, находящиеся между двумя точками в цепи, при вычислении разницы между двумя точками.

    Посмотрите на следующие принципиальные схемы. Батарея во всех случаях одна и та же, меняется только добавление резисторов между точками, отмеченными черными точками.Если бы мы измерили разность потенциалов между двумя точками в этих схемах, мы бы получили одинаковый ответ для всех трех случаев.

    Давайте посмотрим на два параллельно подключенных резистора более внимательно. Когда вы создаете схему, вы используете провода, и вы можете подумать, что измерение напряжения в разных местах на проводах будет иметь значение. Это неправда. Измерение разности потенциалов или напряжения будет отличаться только в том случае, если вы измеряете другой набор компонентов. Все точки на проводах, между которыми нет компонентов цепи, дадут вам одинаковые измерения.

    Все три измерения, показанные на рисунке ниже (т. Е. A – B, C – D и E – F), дадут вам одинаковое напряжение. Различные точки измерения слева не имеют компонентов между собой, поэтому нет изменений в потенциальной энергии. То же самое относится и к разным точкам справа. Когда вы измеряете разность потенциалов между точками слева и справа, вы получите одинаковый ответ.

    Рабочий пример 7: Напряжения I

    Рассмотрим принципиальную схему:

    Какое напряжение на резисторе в показанной цепи?

    Проверьте, что у вас есть и единицы

    У нас есть схема с батареей и одним резистором.Нам известно напряжение на батарее. Мы хотим найти это напряжение на резисторе.

    \ [{V} _ {\ text {battery}} = \ text {2} \ text {V} \]

    Применимые принципы

    Мы знаем, что напряжение на батарее должно быть равно общему напряжению на всех других компонентах схемы.

    \ [{V} _ {\ text {battery}} = {V} _ {\ text {total}} \]

    Есть только один другой компонент схемы — резистор.

    \ [{V} _ {\ text {total}} = {V} _ {1} \]

    Это означает, что напряжение на батарее равно напряжению на резисторе.

    \ begin {align *} {V} _ {\ text {battery}} & = {V} _ {\ text {total}} = {V} _ {1} \\ {V} _ {1} & = \ text {2} \ text {V} \ end {align *}

    Рабочий пример 8: Напряжения II

    Рассмотрим схему:

    Какое напряжение на неизвестном резисторе в показанной цепи?

    Проверьте, что у вас есть и единицы

    У нас есть схема с батареей и двумя резисторами. Нам известно напряжение на батарее и на одном из резисторов.Мы хотим найти это напряжение на резисторе.

    \ begin {align *} {V} _ {\ text {battery}} & = \ text {2} \ text {V} \\ {V} _ {B} & = \ text {1} \ text {V} \ end {align *}

    Применимые принципы

    Мы знаем, что напряжение на батарее должно быть равно общему напряжению на всех других компонентах схемы, включенных последовательно.

    \ [{V} _ {\ text {battery}} = {V} _ {\ text {total}} \]

    Общее напряжение в цепи — это сумма напряжений на отдельных резисторах

    \ [{V} _ {\ text {total}} = {V} _ {A} + {V} _ {B} \]

    Использование зависимости между напряжением на батарее и общим напряжением на резисторах

    \ begin {align *} {V} _ {\ text {battery}} & = {V} _ {\ text {total}} \\\\ {V} _ {\ text {battery}} & = {V} _ {1} + {V} _ {\ text {resistor}} \\ \ text {2} \ text {V} & = {V} _ {1} + \ text {1} \ text {V} \\ {V} _ {1} & = \ text {1} \ text {V} \ end {align *}

    Рабочий пример 9: Напряжения III

    Рассмотрим принципиальную схему:

    Какое напряжение на неизвестном резисторе в показанной цепи?

    Проверьте, что у вас есть и единицы

    У нас есть схема с батареей и тремя резисторами.Нам известно напряжение на батарее и двух резисторах. Мы хотим найти это напряжение на неизвестном резисторе.

    \ begin {align *} {V} _ {\ text {battery}} & = \ text {7} \ text {V} \\ {V} _ {\ text {известное}} & = {V} _ {A} + {V} _ {C} \\ & = \ текст {1} \ текст {V} + \ текст {4} \ текст {V} \ end {align *}

    Применимые принципы

    Мы знаем, что напряжение на батарее должно быть равно общему напряжению на всех других компонентах схемы, включенных последовательно.

    \ [{V} _ {\ text {battery}} = {V} _ {\ text {total}} \]

    Общее напряжение в цепи — это сумма напряжений на отдельных резисторах

    \ [{V} _ {\ text {total}} = {V} _ {B} + {V} _ {\ text {known}} \]

    Использование зависимости между напряжением на батарее и общим напряжением на резисторах

    \ begin {align *} {V} _ {\ text {battery}} & = {V} _ {\ text {total}} \\ {V} _ {\ text {battery}} & = {V} _ {B} + {V} _ {\ text {known}} \\ \ text {7} \ text {V} & = {V} _ {B} + \ text {5} \ text {V} \\ {V} _ {B} & = \ text {2} \ text {V} \ end {align *}

    Рабочий пример 10: Напряжения IV

    Рассмотрим принципиальную схему:

    Какое напряжение на параллельной комбинации резисторов в показанной цепи? Подсказка: остальная часть схемы такая же, как и в предыдущей задаче.

    Быстрый ответ

    Схема такая же, как и в предыдущем примере, и мы знаем, что разница напряжений между двумя точками в цепи не зависит от того, что между ними, поэтому ответ такой же, как указано выше \ ({V} _ {\ text {parallel }} = \ текст {2} \ текст {V} \).

    Проверьте, что у вас есть и единицы — длинный ответ

    У нас есть схема с батареей и пятью резисторами (два последовательно и три параллельно). Нам известно напряжение на батарее и двух резисторах.Мы хотим найти это напряжение на параллельных резисторах \ ({V} _ {\ text {parallel}} \).

    \ begin {align *} {V} _ {\ text {battery}} = \ text {7} \ text {V} \\ {V} _ {\ text {известное}} = \ text {1} \ text {V} + \ text {4} \ text {V} \ end {align *}

    Применимые принципы

    Мы знаем, что напряжение на батарее должно быть равно общему напряжению на всех других компонентах схемы.

    \ [{V} _ {\ text {battery}} = {V} _ {\ text {total}} \]

    Напряжения складываются только алгебраически для последовательно соединенных компонентов.Параллельно подключенные резисторы можно рассматривать как один компонент, который включен последовательно с другими компонентами, а затем можно складывать напряжения.

    \ [{V} _ {\ text {total}} = {V} _ {\ text {parallel}} + {V} _ {\ text {known}} \]

    Использование зависимости между напряжением на батарее и общим напряжением на резисторах

    \ begin {align *} {V} _ {\ text {battery}} & = {V} _ {\ text {total}} \\ {V} _ {\ text {battery}} & = {V} _ {\ text {parallel}} + {V} _ {\ text {known}} \\ \ text {7} \ text {V} & = {V} _ {\ text {parallel}} + \ text {5} \ text {V} \\ {V} _ {\ text {parallel}} & = \ text {2} \ text {V} \ end {align *}

    В отличие от последовательного случая, когда мы добавляем резисторы параллельно, мы создаем еще путей , по которым может течь ток.Делая это, мы на уменьшаем на общее сопротивление цепи!

    Взгляните на диаграмму ниже. Слева у нас та же схема, что и в предыдущем разделе, с батареей и резистором. Амперметр показывает ток \ (\ text {1} \) \ (\ text {A} \). Справа мы добавили второй резистор параллельно первому. Это увеличило количество путей (ответвлений), по которым заряд может проходить через цепь — общее сопротивление уменьшилось. Вы можете видеть, что ток в цепи увеличился.Также обратите внимание, что ток в разных ветвях может быть разным.

    Общее сопротивление ряда параллельных резисторов НЕ является суммой отдельных сопротивлений, поскольку общее сопротивление уменьшается с увеличением количества путей прохождения тока. Общее сопротивление для параллельных резисторов равно:

    . \ [\ frac {1} {{R} _ {P}} = \ frac {1} {{R} _ {1}} + \ frac {1} {{R} _ {2}} + \ ldots \ ]

    Давайте рассмотрим случай, когда у нас есть два резистора, включенных параллельно, и выясним, каким будет конечное сопротивление.Эта ситуация показана на диаграмме ниже:

    Применяя формулу для общего сопротивления, получаем:

    \ begin {align *} \ frac {1} {{R} _ {P}} & = \ frac {1} {{R} _ {1}} + \ frac {1} {{R} _ {2}} + \ ldots \\ & \ text {Резисторов всего два} \\ \ frac {1} {{R} _ {P}} & = \ frac {1} {{R} _ {1}} + \ frac {1} {{R} _ {2}} \\ & \ text {Добавить дроби} \\ \ frac {1} {{R} _ {P}} & = \ frac {1} {{R} _ {1}} \ times \ frac {{R} _ {2}} {{R} _ {2 }} + \ frac {1} {{R} _ {2}} \ times \ frac {{R} _ {1}} {{R} _ {1}} \\ \ frac {1} {{R} _ {P}} & = \ frac {{R} _ {2}} {{R} _ {1} {R} _ {2}} + \ frac {{R} _ {1}} {{R} _ {1} {R} _ {2}} \\ & \ text {Переставить} \\ \ frac {1} {{R} _ {P}} & = \ frac {{R} _ {2} + {R} _ {1}} {{R} _ {1} {R} _ {2} } \\ \ frac {1} {{R} _ {P}} & = \ frac {{R} _ {1} + {R} _ {2}} {{R} _ {1} {R} _ {2} } \\ {R} _ {P} & = \ frac {{R} _ {1} {R} _ {2}} {{R} _ {1} + {R} _ {2}} \ end {выровнять *}

    Для любых двух параллельно подключенных резисторов мы теперь знаем, что

    \ [{R} _ {P} = \ frac {\ text {произведение сопротивлений}} {\ text {сумма сопротивлений}} = \ frac {{R} _ {1} {R} _ {2}} { {R} _ {1} + {R} _ {2}} \]

    Делители тока

    Цель

    Проверьте, что происходит с током и напряжением в последовательных цепях при добавлении дополнительных резисторов.

    Аппарат

    • Аккумулятор

    • Вольтметр

    • Амперметр

    • Провода

    • Резисторы

    Метод

    • Подключите каждую цепь, показанную ниже

    • Измерьте напряжение на каждом резисторе в цепи.

    • Измерьте ток до и после каждого резистора в цепи, а также до и после параллельных ветвей.

    Результаты и выводы

    • Сравните токи через отдельные резисторы между собой.

    • Сравните сумму токов через отдельные резисторы с током до параллельных ветвей.

    • Сравните различные измерения напряжения на параллельных резисторах.

    Рабочий пример 11: Параллельные резисторы I

    Цепь содержит два параллельно включенных резистора. Резисторы имеют значения сопротивления \ (\ text {15} \) \ (\ text {Ω} \) и \ (\ text {7} \) \ (\ text {Ω} \).

    Какое полное сопротивление в цепи?

    Проанализировать вопрос

    Нам говорят, что резисторы в цепи включены в параллельную цепь и что нам нужно рассчитать общее сопротивление.Значения двух резисторов указаны в правильных единицах, Ом.

    Применять соответствующие принципы

    Было показано, что полное сопротивление резисторов, включенных параллельно, является произведением сопротивлений, разделенных на сумму. Мы можем использовать

    \ [{R} _ {P} = \ frac {{R} _ {1} {R} _ {2}} {{R} _ {1} + {R} _ {2}} \]

    У нас всего два резистора, и теперь мы резисторы. В данном случае это:

    \ begin {align *} {R} _ {P} & = \ frac {{R} _ {1} {R} _ {2}} {{R} _ {1} + {R} _ {2}} \\ & = \ frac {\ left (\ text {15} \ text {Ω} \ right) \ left (\ text {7} \ text {Ω} \ right)} {\ text {15} \ text {Ω} + \ text {7} \ text {Ω}} \\ & = \ frac {105 {\ Omega} ^ {2}} {\ text {22} \ text {Ω}} \\ & = \ текст {4,77} \ текст {Ω} \ end {align *}

    Цитировать окончательный результат

    Полное сопротивление параллельно включенных резисторов \ (\ text {4,77} \) \ (\ text {Ω} \)

    Рабочий пример 12: Параллельные резисторы II

    Мы добавляем третий параллельный резистор в конфигурацию (настройку) в предыдущем примере.Дополнительный резистор имеет сопротивление \ (\ text {3} \) \ (\ text {Ω} \).

    Какое полное сопротивление в цепи?

    Проанализировать вопрос

    Нам говорят, что резисторы в цепи включены в параллельную цепь и что нам нужно рассчитать общее сопротивление. Значение дополнительного резистора указано в правильных единицах, Ом.

    Применять соответствующие принципы

    Общее сопротивление резисторов, включенных параллельно, составляет

    . \ [\ frac {1} {{R} _ {P}} = \ frac {1} {{R} _ {1}} + \ frac {1} {{R} _ {2}} + \ ldots \ ]

    У нас есть три резистора, и теперь мы резисторы.В данном случае это:

    \ begin {align *} \ frac {1} {{R} _ {P}} & = \ frac {1} {{R} _ {1}} + \ frac {1} {{R} _ {2}} + \ ldots \\ & \ text {есть три резистора} \\ \ frac {1} {{R} _ {P}} & = \ frac {1} {{R} _ {1}} + \ frac {1} {{R} _ {2}} + \ frac {1 } {{R} _ {3}} \\ & \ text {добавить дроби} \\ \ frac {1} {{R} _ {P}} & = \ frac {1} {{R} _ {1}} \ times \ frac {{R} _ {2} {R} _ {3}} {{R} _ {2} {R} _ {3}} + \ frac {1} {{R} _ {2}} \ times \ frac {{R} _ {1} {R} _ {3} } {{R} _ {1} {R} _ {3}} + \ frac {1} {{R} _ {3}} \ times \ frac {{R} _ {1} {R} _ {2 }} {{R} _ {1} {R} _ {2}} \\ \ frac {1} {{R} _ {P}} & = \ frac {{R} _ {2} {R} _ {3}} {{R} _ {1} {R} _ {2} { R} _ {3}} + \ frac {{R} _ {1} {R} _ {3}} {{R} _ {1} {R} _ {2} {R} _ {3}} + \ frac {{R} _ {1} {R} _ {2}} {{R} _ {1} {R} _ {2} {R} _ {3}} \\ & \ text {переставить} \\ \ frac {1} {{R} _ {P}} & = \ frac {{R} _ {2} {R} _ {3} + {R} _ {1} {R} _ {3} + { R} _ {2} {R} _ {3}} {{R} _ {1} {R} _ {2} {R} _ {3}} \\ {R} _ {P} & = \ frac {{R} _ {1} {R} _ {2} {R} _ {3}} {{R} _ {2} {R} _ {3} + {R} _ {1} {R} _ {3} + {R} _ {2} {R} _ {3}} \\ {R} _ {P} & = \ frac {\ left (\ text {15} \ text {Ω} \ right) \ left (\ text {7} \ text {Ω} \ right) \ left (\ text { 3} \ text {Ω} \ right)} {\ left (\ text {7} \ text {Ω} \ right) \ left (\ text {3} \ text {Ω} \ right) + \ left (\ text {15} \ text {Ω} \ right) \ left (\ text {3} \ text {Ω} \ right) + \ left (\ text {7} \ text {Ω} \ right) \ left (\ text { 15} \ text {Ω} \ right)} \\ {R} _ {P} & = \ frac {315 {\ Omega} ^ {3}} {21 {\ Omega} ^ {2} + 45 {\ Omega} ^ {2} + 105 {\ Omega} ^ { 2}} \\ {R} _ {P} & = \ frac {315 {\ Omega} ^ {3}} {171 {\ Omega} ^ {2}} \\ {R} _ {P} & = \ text {1,84} \ text {Ω} \ end {align *}

    Цитировать окончательный результат

    Полное сопротивление параллельно включенных резисторов \ (\ text {1,84} \) \ (\ text {Ω} \)

    При расчете сопротивления для сложных конфигураций резисторов вы можете начать с любой комбинации двух резисторов (последовательно или параллельно) и рассчитать их общее сопротивление.Затем вы можете заменить их одним резистором с рассчитанным вами общим сопротивлением. Теперь используйте этот новый резистор в сочетании с любым другим резистором и повторяйте процесс, пока не останется только один резистор. В приведенном выше примере мы могли бы просто использовать ответ из первого примера параллельно с новым резистором, и мы получили бы тот же ответ.

    Рабочий пример 13: Параллельные резисторы III

    Мы добавляем третий параллельный резистор к первой конфигурации (настройке), работающей параллельно.Дополнительный резистор имеет сопротивление \ (\ text {3} \) \ (\ text {Ω} \)

    Какое полное сопротивление в цепи?

    Проанализировать вопрос

    Нам говорят, что резисторы в цепи включены в параллельную цепь и что нам нужно рассчитать общее сопротивление. Значение дополнительного резистора указано в правильных единицах, Ом.

    Применять соответствующие принципы

    Мы можем поменять местами резисторы, не меняя схему:

    Мы уже рассчитали, что полное сопротивление двух резисторов в пунктирной рамке равно \ (\ text {4,77} \) \ (\ text {Ω} \). {2}} {\ text {11,77} \ Omega} \\ & = \ текст {1,84} \ текст {Ω} \ end {align *}

    Цитировать окончательный результат

    Полное сопротивление параллельно включенных резисторов равно \ (\ text {1,84} \) \ (\ text {Ω} \).Это тот же результат, что и при одновременном сложении всех трех резисторов.

    Высокие оценки в науке — залог вашего успеха и будущих планов. Проверьте себя и узнайте больше о практике Сиявулы.

    Зарегистрируйтесь и проверьте себя

    Сопротивление

    Учебное пособие Упражнение 17.1

    Как называется единица сопротивления и каков ее символ?

    Решение пока недоступно

    Объясните, что происходит с общим сопротивлением цепи, когда резисторы добавляются последовательно?

    Решение пока недоступно

    Объясните, что происходит с общим сопротивлением цепи, когда резисторы добавляются параллельно?

    Решение пока недоступно

    Почему батарейки разряжаются?

    Решение еще не доступно

    Типы, применение, детали »Электроника

    Резисторы

    являются одними из наиболее широко используемых компонентов в электронных схемах — существует множество различных типов резисторов, имеющих разные свойства и используемых по-разному в разных схемах.


    Resistor Tutorial:

    Resistors Обзор Углеродный состав Карбоновая пленка Металлооксидная пленка Металлическая пленка Проволочная обмотка SMD резистор MELF резистор Переменные резисторы Светозависимый резистор Термистор Варистор Цветовые коды резисторов Маркировка и коды SMD резисторов Характеристики резистора Где и как купить резисторы Стандартные номиналы резисторов и серия E


    Резисторы всех типов в большом количестве используются в производстве электронного оборудования.Фактически, резистор, вероятно, является наиболее распространенным типом электронного компонента, используемого в электрических и электронных схемах.

    Существует большое количество различных типов резисторов, которые можно купить и использовать. Свойства этих разных резисторов различаются, и это помогает получить резистор правильного типа для любой конкретной конструкции, чтобы гарантировать получение наилучших характеристик.

    Хотя многие резисторы будут работать в различных приложениях, тип резистора может быть важен в некоторых случаях.Соответственно, необходимо знать о различных типах резисторов и о том, в каких приложениях можно использовать каждый тип резистора.

    Выбор резисторов с постоянными выводами или различных типов

    Что такое резистор?

    Резисторы

    используются практически во всех электронных схемах и многих электрических. Резисторы, как следует из их названия, противостоят току электричества, и эта функция является ключевой для работы большинства цепей.

    Примечание о сопротивлении:

    Сопротивление — один из ключевых факторов, используемых в электрических и электронных схемах.Сопротивление — это свойство материалов сопротивляться потоку электричества, и оно регулируется законом Ома.

    Подробнее о Сопротивление.

    Для резисторов используются два основных символа схемы. Самый старый из них до сих пор широко используется в Северной Америке и состоит из зубчатой ​​линии, обозначающей провод, используемый в резисторе. Другой символ цепи резистора представляет собой небольшой прямоугольник, который часто называют международным символом резистора, и он более широко используется в Европе и Азии.

    Символы цепи резистора

    Единицей измерения или сопротивления является Ом, Ом, а значения резистора могут быть указаны в единицах Ом — Ом, тысячи Ом или килом — кОм и миллионы Ом, мегом, МОм. При написании на схемах таких значений, как 10 кОм, можно увидеть, что это означает 10 кОм или 10 кОм. Знак Омега часто опускается, а десятичная точка заменяется множителем: например, 1R5 будет 1,5 Ом, 100R — 100 Ом, 4k7 — 4,7 кОм, 2M2 — 2,2 МОм и т. Д.

    Есть много разных типов резисторов.Некоторые из них предназначены для специальных применений, таких как использование в качестве переменных резисторов, а другие используются для ограничения перенапряжения, в то время как другие обеспечивают переменное сопротивление в зависимости от температуры. Все эти характеристики можно использовать.

    Однако для постоянных резисторов необходимо учитывать разные характеристики.

    Несмотря на то, что фактическое сопротивление компонента имеет первостепенное значение, необходимо учитывать и другие характеристики. Рассеиваемая мощность, шум, индуктивность, термическая стабильность и ряд других характеристик могут влиять на работу цепи, в которой используется резистор.

    Различные материалы и структура резистора могут иметь большое влияние. Соответственно, при выборе резистора, который будет использоваться, эти характеристики также должны быть приняты во внимание.

    Принципиальное различие типов резисторов

    Первые основные категории, к которым могут быть отнесены различные типы резисторов, — фиксированные или переменные. Эти разные типы резисторов используются для разных приложений:

    • Фиксированные резисторы: Фиксированные резисторы на сегодняшний день являются наиболее широко используемым типом резисторов.Они используются в электронных схемах для установки правильных условий в цепи. Их значения определяются на этапе проектирования схемы, и их никогда не следует изменять для «настройки» схемы. Существует множество различных типов резисторов, которые можно использовать в различных обстоятельствах, и эти различные типы резисторов более подробно описаны ниже.
    • Переменные резисторы: Эти резисторы состоят из фиксированного резисторного элемента и ползунка, который подключается к основному резистивному элементу.Это дает три соединения с компонентом: два соединены с фиксированным элементом, а третье — с ползунком. Таким образом, компонент действует как переменный делитель потенциала, если используются все три соединения. Можно подключить к ползунку и одним концом, чтобы обеспечить резистор с переменным сопротивлением.
      Потенциометр предварительной настройки для углеродной пленки Переменные резисторы и потенциометры широко используются для всех форм управления: — от регуляторов громкости на радиоприемниках и ползунков в аудиомикшерах до множества областей, где требуется переменное сопротивление.
      Потенциометр и переменный резистор Строго говоря, потенциометр — это компонент, в котором есть фиксированный резистор, который имеет ползунок для обеспечения деления потенциала от напряжения наверху. Переменный резистор фактически такой же, но с ползунком, соединенным с одним концом резистора, так что он обеспечивает истинное переменное сопротивление.

    Типы постоянного резистора

    Есть несколько различных типов постоянного резистора:

    • Состав углерода: Резистор углеродного состава — это тип резистора, который когда-то был очень распространен — ​​это был основной тип резистора, но теперь редко используется, потому что новые формы резистора обеспечивают лучшую производительность, они меньше и тоже дешевле.

      Резисторы из углеродного состава получают путем смешивания гранул углерода со связующим, которое затем превращается в небольшой стержень. Этот тип резистора был большим по сегодняшним стандартам и имел большой отрицательный температурный коэффициент.
      Резисторы также страдали от больших и беспорядочных необратимых изменений сопротивления в результате нагрева или старения. В дополнение к этому гранулированный характер углерода и связующего приводит к возникновению высокого уровня шума при протекании тока.


    • Углеродная пленка: Этот тип резистора был представлен на заре транзисторной технологии, когда уровни мощности имели тенденцию к снижению.
      Карбоновый пленочный резистор Углеродный пленочный резистор формируется путем «крекинга» углеводорода на керамическом каркасе. Сопротивление полученной осажденной пленки устанавливали путем врезания спирали в пленку. Это сделало эти резисторы очень индуктивными и мало пригодными для многих ВЧ-приложений. Они показали температурный коэффициент от -100 до -900 частей на миллион на градус Цельсия. Углеродная пленка защищена либо конформным эпоксидным покрытием, либо керамической трубкой.
    • Металлооксидный пленочный резистор: Этот тип резистора в настоящее время является наиболее широко используемой формой резистора.Вместо углеродной пленки в этом типе резисторов используется пленка оксида металла, нанесенная на керамический стержень. Как и в случае с углеродной пленкой, сопротивление можно регулировать, вырезая в пленке спиральную канавку. Пленка снова защищена конформным эпоксидным покрытием. Этот тип резистора имеет температурный коэффициент около + или — 15 частей на миллион на градус Цельсия, что дает ему намного лучшие характеристики по сравнению с любым резистором на углеродной основе. Кроме того, этот тип резистора может поставляться с гораздо меньшим допуском, стандартным является 5% или даже 2%, а доступны версии с 1%.Они также демонстрируют гораздо более низкий уровень шума, чем углеродные резисторы, однако в основном они были заменены металлическими пленочными резисторами.
    • Металлический пленочный резистор: Металлический пленочный резистор очень похож на металлооксидный пленочный резистор. Визуально он очень похож, и производительность также сопоставима. Вместо металлооксидной пленки в этом типе резистора используется металлическая пленка, как следует из названия. Могут использоваться такие металлы, как никелевый сплав.
      Металлопленочный резистор с выводами Металлопленочный резистор — это тип, который наиболее широко используется, когда требуется резистор с выводами.
    • Резистор с проволочной обмоткой: Этот тип резистора обычно предназначен для приложений с большой мощностью. Эти резисторы изготавливаются путем наматывания на каркас провода с более высоким, чем обычно, сопротивлением (провод сопротивления).

      Более дорогие разновидности наматываются на керамический каркас и могут быть покрыты стекловидной или силиконовой эмалью.Этот тип резистора подходит для высоких мощностей и демонстрирует высокий уровень надежности при высоких мощностях наряду со сравнительно низким уровнем температурного коэффициента, хотя это будет зависеть от ряда факторов, включая первый, используемый провод и т. Д. В качестве резисторов с проволочной обмоткой часто предназначены для применения с высокой мощностью, некоторые разновидности спроектированы таким образом, что их можно установить на радиаторе, чтобы гарантировать, что мощность рассеивается в металлоконструкциях, чтобы ее можно было унести.

      Ввиду того, что они намотаны, они не подходят для работы на частотах выше низких, хотя, если намотать части резистивного провода в разные стороны, индуктивность можно несколько уменьшить.


    • Резисторы для поверхностного монтажа: Технология поверхностного монтажа, SMT в настоящее время является основным форматом, используемым для электронных компонентов. Их проще использовать в автоматизированном производстве, и они способны обеспечить очень высокий уровень производительности. В резисторах SMT используются технологии, аналогичные другим формам, но в формате для поверхностного монтажа.

    Другие типы резисторов

    Хотя большинство резисторов представляют собой стандартные постоянные резисторы или переменные резисторы, существует ряд других типов резисторов, которые используются в более узких или специализированных приложениях.

    • Светозависимый резистор / фоторезистор: Светозависимые резисторы или фоторезисторы изменяют свое сопротивление в зависимости от уровня освещенности. Они используются в ряде сенсорных приложений и во многих случаях представляют собой очень экономичное решение.

      Типичный светодиодный резистор, зависимый от света Светозависимые резисторы имеют задержку во времени, необходимом для реакции на изменение освещенности, но они дешевы и просты в использовании.


    • Термистор: Как видно из названия, термисторы являются термочувствительными резисторами.Сопротивление термистора зависит от температуры. Некоторые имеют отрицательный температурный коэффициент, термисторы NTC, другие имеют положительный температурный коэффициент, термисторы PTC.
    • Варистор: Варисторы доступны в нескольких формах. По сути, эти электронные компоненты изменяют свое сопротивление в зависимости от приложенного напряжения, и в результате они находят применение для защиты от скачков напряжения и перенапряжения. Часто их можно увидеть как Movistors, что является сокращением слов M etal O xide V ar istor .

      Выбор варисторов с выводами Варисторы — это устройства, которые широко используются в удлинителях сети с защитой от перенапряжения или переходных процессов и используются для защиты компьютеров. Следует помнить, что каждый раз, когда варистор получает импульс, его свойства незначительно меняются.


    Хотя резисторы можно рассматривать как простые электронные компоненты для использования, существует ряд параметров, которые необходимо учитывать при выборе правильного типа резистора.Важны не только сопротивление, но и параметры. Выдерживаемое напряжение, рассеиваемая мощность и тип самого резистора — все это влияет на производительность. Поскольку имеется множество типов резисторов, необходимо выбирать правильный тип для каждого конкретного применения. Таким образом можно гарантировать лучшую производительность.

    Другие электронные компоненты:
    Резисторы Конденсаторы Индукторы Кристаллы кварца Диоды Транзистор Фототранзистор Полевой транзистор Типы памяти Тиристор Разъемы Разъемы RF Клапаны / трубки Аккумуляторы Переключатели Реле
    Вернуться в меню «Компоненты».. .

    Факты о резисторах для детей

    Два резистора в последовательной цепи Два резистора в параллельной цепи

    Резистор ограничивает электрический ток, протекающий по цепи. Сопротивление — это ограничение тока. В резисторе энергия электронов, проходящих через резистор, изменяется на тепло и / или свет. Например, в лампочке есть резистор из вольфрама, который преобразует электроны в свет.

    Последовательный и параллельный

    Резисторы могут быть соединены в различных комбинациях для создания цепи:

      Серия
    1. — где резисторы соединены один за другим .
    2. Параллельно — где резисторы соединены друг с другом .

    Есть много разных типов резисторов. Резисторы имеют разные номиналы, чтобы сообщить электрикам, с какой мощностью они могут выдержать до того, как сломаются, и насколько точно они могут замедлить поток электричества.Последовательное соединение двух резисторов приводит к более высокому сопротивлению, чем при параллельном подключении тех же двух резисторов. Чтобы резистор не достиг своей емкости, разместите резисторы параллельно, чтобы общее сопротивление было ниже. В настоящее время в электротехнической промышленности во многих случаях используются резисторы на основе так называемой технологии поверхностного монтажа, которые могут быть очень маленькими.

    Расчет сопротивления

    • Последовательная цепь: Rt = R1 + R2 + R3 + R4 … Rn
    • Параллельная цепь: 1 / Rt = 1 / R1 + 1 / R2 + 1 / R3…1 / Rn

    Где R — номинал резистора

    Закон Ома

    Формула закона Ома, V = I * R, утверждает, что падение напряжения на компоненте равно произведению тока, протекающего в компоненте, на сопротивление компонента. Используя закон Ома, вы можете изменить формулу, если необходимо, чтобы найти другой результат: I = V / R или R = V / I

    Код цвета

    Номиналы резистора

    указаны по цветам, указанным на его стороне.Цветные полосы, которые используются на сторонах резистора, являются черными, коричневыми, красными, оранжевыми, желтыми, зелеными, синими, пурпурными, серыми и белыми. Каждый цвет представляет собой разное число. Черная полоса представляет собой цифру 0, коричневая полоса представляет собой цифру 1, красная — 2 и так далее, вплоть до белой, которая представляет собой цифру 9. Эти числа очень важны в электронной области.

    На стороне резистора может быть несколько цветных полос. У наиболее распространенных — четыре, но их может быть до 6 на резистор.На четырехполосном резисторе последняя полоса золотая или серебряная. Золотая полоса представляет собой положительный или отрицательный 5% допуск. Серебряная полоса на резисторе соответствует положительному или отрицательному допуску в 10%. Держите эту полосу с правой стороны и читайте цвета слева направо. Первые две полосы читаются как числа, которые они представляют в цветовом коде. Третья полоса действует как множитель для других полос, поэтому, например, если третья полоса была оранжевой полосой, которая равна 3, это будет означать, что вы умножаете два числа на 1000.Короче говоря, вы добавляете значение цвета нулями в конце, поэтому добавляете три нуля.

    Приложения

    Резисторы

    используются по-разному. Прежде всего, они вставлены в цепи, чтобы защитить компоненты от повреждений, например светодиоды. Они также контролируют количество тока, протекающего в цепи, например, если вы хотите, чтобы ток был замедлен, вы должны добавить больше резисторов, чтобы создать большее сопротивление в цепи. Резисторы также могут распределять напряжение между различными частями цепи и управлять временной задержкой.

    Материалы резисторов

    Вы можете найти множество различных типов резисторов. Все они изготовлены из резистивного материала, заключенного в корпус из непроводящего материала, например из пластика. Постоянные резисторы обычно изготавливаются из углерода, заключенного в пластиковый цилиндр, с соединительным проводом на обоих концах. Большинство резисторов, используемых сегодня в электронике, представляют собой углеродные резисторы. Старые резисторы были сделаны из металла с плохой проводимостью, чтобы ограничить прохождение электричества.

    Изображения для детей

    • Резисторы осевые на ленте.Компонент вырезается из ленты во время сборки, и деталь вставляется в плату.

    • Силовой резистор в алюминиевом корпусе мощностью 50 Вт с теплоотводом

    • Резистор силовой ВЗР 1,5кОм 12Вт, изготовлен в 1963 году в Советском Союзе

    • Комплект одинарных резисторов (SIL) с 8 отдельными резисторами по 47 Ом. Один конец каждого резистора подключен к отдельному выводу, а другие концы все вместе подключены к оставшемуся (общему) выводу — выводу 1, конец которого обозначен белой точкой.

    • Резисторы с выводами для сквозного монтажа

    • Три резистора из углеродного состава в радиоприемнике 1960-х годов (электронная лампа)

    • Углеродный пленочный резистор с открытой углеродной спиралью (Tesla TR-212 1 кОм)

    • Угольный резистор, напечатанный непосредственно на контактных площадках SMD на печатной плате. Внутри органайзера Psion II

      года выпуска 1989 года
    • Прецизионная сеть тонкопленочных резисторов с лазерной обрезкой от Fluke, используемая в мультиметре Keithley DMM7510.Керамическая основа со стеклянной герметичной крышкой.

    • Проволочные резисторы большой мощности для динамического торможения вагонов с электроприводом. Такие резисторы могут рассеивать много киловатт в течение длительного периода времени.

    • Типовой потенциометр для монтажа на панели

    • Чертеж потенциометра с вырезом корпуса, показывающий детали: ( A ) вал, ( B ) неподвижный резистивный элемент из углеродной композиции, ( C ) грязесъемник из фосфористой бронзы, ( D ) вал, прикрепленный к дворнику, ( E, G ) клеммы, подключенные к концам резистивного элемента, клеммы ( F ), подключенные к дворнику.

    • Десятилетний ящик сопротивления «Kurbelwiderstand», произведенный в бывшей Восточной Германии.

    • На этом изображении показаны четыре резистора для поверхностного монтажа (компонент в верхнем левом углу — конденсатор), включая два резистора с нулевым сопротивлением. Вместо проводных перемычек часто используются перемычки с нулевым сопротивлением, поэтому их можно вставить с помощью машины для вставки резисторов. Их сопротивление ненулевое, но незначительное.

    Что такое резистор? Конструкция, принципиальная схема и применение

    Резистор — один из наиболее важных электрических и электронных компонентов, используемых в различных электронных устройствах.Они доступны в различных размерах и формах на рынке в зависимости от области применения. Мы знаем, что любая базовая электрическая и электронная схема работает с протеканием тока. Кроме того, он также подразделяется на два типа, а именно проводники и изоляторы . Основная функция проводника — пропускать ток, тогда как изолятор не пропускает ток. Всякий раз, когда высокое напряжение подается через металлический проводник, через него проходит полное напряжение.Если резистор подключен к этому проводнику, то поток тока, а также напряжение будут ограничены. В этой статье обсуждается обзор резистора.


    Что такое резистор?

    Определение для резистора таково: это базовый двухконтактный электрический и электронный компонент , используемый для ограничения тока в цепи. Сопротивление потоку тока приведет к падению напряжения. Эти устройства могут обеспечивать постоянное регулируемое значение сопротивления.Величину резисторов можно выразить в Ом.

    Резистор

    Резисторы используются в нескольких электрических, а также в электронных схемах , чтобы получить известное падение напряжения, в противном случае отношение тока к напряжению (C-to-V). Когда ток в цепи идентифицируется, можно использовать резистор для создания идентифицированной разности потенциалов, которая пропорциональна току. Точно так же, если падение напряжения в двух точках в цепи идентифицировано, резистор может использоваться для создания идентифицированного тока, который пропорционален этой несходственности.Пожалуйста, перейдите по ссылке, чтобы узнать больше о:

    Символ резистора

    Что такое сопротивление?

    Сопротивление может зависеть от закона Ома , открытого немецким физиком, а именно « Георг Симон ».

    Закон Ома

    Закон Ома можно определить как ; напряжение на резисторе прямо пропорционально току, протекающему через него. Уравнение закона Ома:

    В = I * R

    Где «V» — напряжение, «I» — ток, а «R» — сопротивление

    Единицы измерения сопротивления — Ом, а несколько более высоких значений Ом включают в себя КОм (килоом), МОм (мегаом). , Милли Ом и т. Д.

    Конструкция резистора

    Например, углеродный пленочный резистор используется для описания конструкции резистора .Конструкция резистора показана на схеме ниже. Этот резистор состоит из двух выводов, как обычный резистор. Конструкция углеродного пленочного резистора может быть выполнена путем размещения углеродного слоя на керамической подложке. Углеродная пленка представляет собой резистивный материал по отношению к прохождению тока в этом резисторе. Однако он блокирует некоторое количество тока.

    Конструкция углеродного пленочного резистора

    Керамическая подложка действует как изолирующий материал по отношению к току. Таким образом, он не пропускает тепло через керамику.Таким образом, эти резисторы могут без вреда выдерживать высокие температуры. Торцевые заглушки резистора металлические, они размещаются на обоих концах выводов. Две клеммы подключены к двум металлическим торцевым крышкам на резисторе.

    Резистивный элемент этого резистора покрыт эпоксидной смолой, предназначенной для обеспечения безопасности. Эти резисторы в основном используются из-за меньшего шума, который они производят по сравнению с резисторами из углеродного состава. Допустимые отклонения этих резисторов ниже, чем у резисторов из углеродистой стали.Значение допуска может быть определено как несходство между нашим предпочтительным значением сопротивления и истинным значением конструкции. Доступны резисторы в диапазоне от 1 Ом до 10 МОм.

    В этом резисторе предпочтительное значение сопротивления может быть достигнуто путем обрезания толщины углеродного слоя по спирали в зависимости от его длины. Как правило, это можно сделать с помощью LASER . Как только необходимое значение сопротивления будет достигнуто, резка металла будет остановлена.

    В резисторах этого типа, когда сопротивление этих резисторов уменьшается при повышении температуры, что известно как высокий отрицательный температурный коэффициент.

    Схема резистора

    Схема простого резистора показана ниже. Эта схема может быть спроектирована с использованием резистора, батареи и светодиода. Мы знаем, что функция сопротивления заключается в ограничении прохождения тока через компонент.

    Схема резистора

    В следующей схеме, если мы хотим соединить светодиод напрямую с батареей источника напряжения, он немедленно выйдет из строя.Поскольку светодиод не пропускает через него большое количество тока, по этой причине между батареей и светодиодом используется резистор для управления потоком тока к светодиоду от батареи.

    Значение сопротивления в основном зависит от номинала батареи. Например, если номинал батареи высокий, то мы должны использовать резистор с высоким значением сопротивления. Величину сопротивления можно измерить по формуле закона Ома.

    Например, номинальное напряжение светодиода составляет 12 вольт, а номинальное значение тока — 0.1 А, иначе 100 мА, затем рассчитайте сопротивление по закону Ома.

    Мы знаем, что Закон Ома V = I X R

    Из приведенного выше уравнения сопротивление можно измерить как R = V / I

    R = 12 / 0,1 = 120 Ом

    Итак, в приведенной выше схеме используется резистор 120 Ом, чтобы избежать повреждения светодиода из-за перенапряжения батареи.

    Последовательные и параллельные резисторы

    Простой способ подключения резисторов последовательно и параллельно в цепи обсуждается ниже.

    Резисторы в последовательном соединении

    При последовательном соединении цепи, когда резисторы включены последовательно в цепь, ток через резисторы будет одинаковым. Напряжение на всех резисторах эквивалентно количеству напряжений на каждом резисторе. Принципиальная схема последовательно соединенных резисторов представлена ​​ниже. Здесь резисторы, используемые в схеме, обозначены R1, R2, R3. Суммарное сопротивление трех резисторов можно записать как

    .

    R Итого = R1 + R2 = R3

    Резисторы в последовательном соединении

    Резисторы в параллельном соединении

    В соединении параллельной цепи , когда резисторы включены в цепь параллельно, тогда напряжение на всех резисторах будет одинаковым.Поток тока через три компонента будет таким же, как величина тока через каждый резистор.

    Принципиальная схема резисторов , подключенных параллельно , показана ниже. Здесь резисторы, используемые в схеме, обозначены R1, R2 и R3. Суммарное сопротивление трех резисторов можно записать как

    .

    R Итого = R1 + R2 = R3

    1 / R Итого = 1 / R1 + 1 / R2 + 1 / R3.

    В результате Rtotal = R1 * R2 * R3 / R1 + R2 + R3

    Резисторы в параллельном соединении

    Расчет значения сопротивления

    Значение сопротивления резистора можно рассчитать с помощью следующих двух методов

      • Расчет значения сопротивления с использованием цветового кода
    • Расчет значения сопротивления с помощью мультиметра
    Расчет значения сопротивления с использованием цветового кода

    Значение сопротивления резистора можно рассчитать с помощью цветовых полос резистора.Перейдите по этой ссылке, чтобы узнать о различных типах резисторов и расчетах их цветовых кодов в электронике.

    Цветовой код резистора
    Расчет значения сопротивления с помощью мультиметра

    Пошаговая процедура вычисления сопротивления резистора с помощью мультиметра обсуждается ниже.

    Мультиметр
      • Второй способ расчета сопротивления можно сделать с помощью мультиметра или омметра. Основное назначение мультиметра — вычисление трех функций, таких как сопротивление, ток и напряжение.
      • Мультиметр состоит из двух щупов, таких как черный халат и красный халат.
      • Вставьте черный щуп в COM-порт, а красный щуп — в VΩmA на мультиметре.
      • Сопротивление резистора можно рассчитать с помощью двух разных щупов мультиметра.
      • Перед вычислением сопротивления вы должны поместить круглый диск в направлении ома, которое указано на мультиметре символом Ом (Ом).

    Применение резистора

    Применения резистора включают следующее.

      • Высокочастотные инструменты
      • Модуляторы и демодуляторы
      • Усилители обратной связи

    Таким образом, это все об обзоре резистора, который включает в себя, что такое резистор, что такое сопротивление, конструкция резистора, схема резистора, резисторы, включенные последовательно и параллельно, расчет значения сопротивления и приложения.Вот вам вопрос, в чем преимущества резистора ?

    Как работает резистор?

    Как работает резистор?

    Многие из ваших проектов MakeCrate требуют в схемах резисторов разной силы, поэтому вам может быть интересно, что делает резистор, как они работают и что произойдет, если вы их не укажете.

    Наша статья «Как работает светодиод?» представила концепцию проводника — материала, который легко переносит электричество — и изолятора — материала, препятствующего прохождению электричества.Разные материалы могут проводить электричество легче, чем другие. Иногда мы хотим уменьшить поток электричества, и использование материалов, замедляющих поток, может помочь в этом. Здесь на помощь приходят резисторы.

    Давайте начнем со словаря:

    Электричество — это поток электронов с, которые являются отрицательно заряженными частицами.

    Напряжение — это разница зарядов между двумя точками в цепи. Напряжение — это также силы электрического тока.

    Ток измеряет, насколько быстро движутся электроны.

    Простой способ понять разницу между напряжением и током — это представить резервуар для воды, сливаемый через шланг, прикрепленный к отверстию в дне. Здесь вода действует как поток электричества. Напряжение — это величина давления воды, а сила тока — это скорость потока воды.

    Сопротивление измеряет способность материала сопротивляться потоку электронов через него.

    Если ширина шланга внизу остается прежней, то при добавлении большего количества воды в бак вода по шлангу течет быстрее. Точно так же добавление большего напряжения к цепи увеличивает ток.

    Однако, если шланг заменить шлангом меньшего размера, то же давление воды приведет к более медленному течению воды. Размер шланга обеспечивает сопротивление и снижает ток. Резистор в цепи играет ту же роль, уменьшая ток через компоненты, с которыми они используются.

    Как резисторы сопротивляются?

    Резисторы в проектах MakeCrate — это проходные резисторы , разработанные с двумя гибкими концами для сгибания и вставки в макетную плату. Сердечник резистора состоит из спирали или спирали из проводящего материала, обернутой вокруг изолирующего сердечника. Материал очень и очень тонкий, что заставляет ток замедляться, обеспечивая сопротивление. Чтобы увеличить сопротивление, можно увеличить количество петель спирали.

    (изображение любезно предоставлено http://electronics.stackexchange.com/questions/17826/whats-inside-a-resistor)

    Что произойдет, если я не использую резистор?

    Если ток, проходящий через компонент, слишком велик для того, чтобы компонент мог с ним справиться, он может перегреть компонент и, возможно, повредить его, или это может вызвать повреждение вашего микроконтроллера (Arduino). Светодиоды быстро перегорят, если не использовать надлежащее сопротивление.

    Если вы хотите безопасно увидеть эффект, который резистор может оказывать на схему, создайте схему, как в вашем проекте «Make Some Noise», но вместо подключения зуммера к земле с помощью провода используйте резистор.Вы увидите, что уменьшение тока приводит к тому, что зуммер становится намного тише.

    Чтобы узнать, как определить сопротивление конкретного резистора, см. Сообщение здесь.

    Для получения дополнительной информации о резисторах ознакомьтесь с этой статьей в Интернете.

    САМАЯ ПРОСТАЯ ЦЕПЬ:

    САМАЯ ПРОСТАЯ ЦЕПЬ:

    ЗАКОН ОМА

    САМАЯ ПРОСТАЯ ЦЕПЬ:

    Мы можем заставить ток течь по кругу (цепи) соединяя клеммы аккумулятора вместе.Это растопит провод, разжигай искры и, возможно, разжигай огонь, так что не делай этого. Вместо этого подключите что-нибудь для управления током. Способность к управляющий ток называется сопротивлением, и все материалы имеют его в некоторой степени — на самом деле мы классифицируем материалы в соответствии с их сопротивление: с очень низким сопротивлением — проводники, с много сопротивления — изоляторы. Есть устройства, называемые резисторами которые используются в электронных гаджетах — у них есть сопротивление, которое что-то среднее между проводимостью и изоляцией и предсказуемо.Так вот безопасная цепь:

    Батарея имеет определенное нажатие, называется электродвижущей силой или ЭДС. Это измеряется в единицах, называемых вольт . Обозначаем ЭДС (часто называемую напряжением) в формулах буквой E . Напряжение необходимо измерять между двумя точками цепи в таким же образом измеряется высота между двумя точками на сторона горы. Нет такого понятия, как «0 вольт», за исключением того, что напряжение между двумя точками равно 0, если они подключены вместе.

    Резистор имеет определенную величину Сопротивление, измеренное в единицах, называется Ом . Мы указываем сопротивление в формулах с буквой R .

    Когда ток течет, мы измеряем его в единицы называют ампер , и обозначают его буквой I .

    Эти три связаны простой формулой называется законом Ома:

    I = E / R

    Также написано E = IR или R = E / I.

    Это говорит нам ток, если мы знаем напряжение и сопротивление, или напряжение, если мы знаем ток и сопротивление, или сопротивление, если мы знаем ток и напряжение. Если вам это кажется немного замкнутым, вы правы. Мы можем измерить ток силой магнитного поля, которое он будет генерировать, но там не является критерием для измерения напряжения, кроме как увидеть, сколько тока течет через известное сопротивление. А как узнать сопротивление? Мы применяем известное напряжение и посмотрите, сколько тока течет.

    Определение единиц также круглое: 1 ампер — это количество тока, протекающего через резистор сопротивлением 1 Ом. если приложено 1 вольт.


    Два резистора последовательно:

    Будет немного сложнее, если есть два резистора:

    Какой бы ни был ток, он такой же в A, B и C. (Больше некуда течь ток).

    Напряжение между A и C равно этому между A и B добавляется к тому, что между B и C.

    E AC = E AB + E BC

    Напряжения складываются, как и высота дом — это сумма высот его этажей.

    Напряжение на каждом резисторе равно пропорционально сопротивлению каждого резистора.

    E AB / R 1 = E BC / R 2

    Видите ли, закон Ома верен для каждой части схема, а также схема в целом.Какой бы ток ни тек, это то же самое в каждом резисторе, поэтому напряжения будут регулироваться самих себя.

    Суммарное сопротивление R 1 + R 2


    Два параллельных резистора

    Ток через А равен току через B плюс ток через C. Ток разделяется и идет вместе, как вода, текущая вокруг острова.

    Напряжение на R 1 такое же, как на напряжение на R 2.

    E AB = E AC, так I B R 1 = I C R 2 и I B / R 2 = I C / R 1

    Другими словами, ток через каждый резистор обратно пропорционален номиналам резисторов. Также важно помнить резистор высокого номинала проходит через небольшой Текущий.

    Мы можем решить вышеуказанную проблему для полного тока ( I B + I C) и получите эквивалент сопротивление для двух резисторов:

    В частном случае, когда резисторы То же, эквивалентное сопротивление — R 1 /2. Это появляется чаще, чем вы можно было ожидать.

    В другом особом случае, когда R2 больше, чем 100-кратное значение R1, R2 составляет такую ​​небольшую часть ток, который мы не удосужились включать в расчеты. Тогда мы говорим, что R2 не загружает схему.


    Вот более сложный пример:

    R1 — резистор особого типа. с регулируемым краном посередине.Это действительно заставляет R1 вести себя как два последовательно включенных резистора. Если мы скажем, что R2 в 100 раз больше R1, мы можем оставьте это в расчетах и ​​обнаружите, что напряжение E2 будет напрямую зависит от положения крана.

    Если бы R2 было сопоставимо с R1 по стоимости, мы бы нужно вычислить, сначала решив R2 и нижнюю часть R1 как два резистора параллельно, и используя результат этого последовательно расчет, чтобы найти напряжение E2 и полный ток.

    Добавить комментарий

    Ваш адрес email не будет опубликован. Обязательные поля помечены *